Qureshi University, Advanced courses, via cutting edge technology, News, Breaking News | Latest News And Media | Current News
admin@qureshiuniversity.com

Admissions | Accreditation | Booksellers | Catalog | Colleges | Contact Us | Continents/States/Districts | Contracts | Distance Education | Emergency | Examinations | Forms | Grants | Hostels | Honorary Doctorate degree | Investment | Instructors | Lecture | Librarians | Membership | Professional Examinations | Programs | Progress Report | Recommendations | Research Grants | Researchers | Students login | School | Search | Seminar | Study Center/Centre | Sponsorship | Tutoring | Thesis | Universities | Work counseling

Pain
Pain Evaluation and Treatment
What should a physician know about pain evaluation and treatment?
Annotation or definition.
Causes.
Classification of pain.
History relevant to pain.
Pain as per location of human body or cause.
Pain Pathway
Severity assessment of pain.
Healthcare setting for treatment.
Medication relevant to cause of pain (analgesics/pain relievers).
Quiz relevant to topic.
Physicians’ documentation on record.

Pain as per location of human body or cause.
What are examples of pain as per location of human body or cause?
Abdominal Pain.
Acute trauma pain.
Back pain.
Burns and associated pain.
Chest pain.
Compressed nerve (pinched nerve).
Extremities pain.
Headache.
Joint Pain.
Mouth pain.
Pelvic Pain.
Post-surgery pain.
Psychogenic pain.
Tooth pain.

Classification of pain.
If there are so many symptoms, medical complaints, or medical problems, why has emphasis been put on pain of a human being?

Pain is the most common symptom, medical complaint, or problem in various health care setting.
Pain can be treated by an individual himself or herself.
Pain may need consultation of a physician in OPF setting or a hospital.

How do you know about classifications of pain?
What are categories of pain?

1. Acute pain and chronic pain.
2. Pain is most often classified by the kind of damage that causes it. The two main categories are pain caused by tissue damage, also called nociceptive pain, and pain caused by nerve damage, also called neuropathic pain.
3. Pain is also classified by the location on part of the body or cause.
4. Psychogenic pain (stress)
5. Mixed

Pain Treatment

Self-treatment.

What types of pain need self-treatment?

Physicians’ consultation.

What type of pain needs physician consultation?

In-hospital treatment with physician’s consultation.

What type of pain needs in-hospital treatment?

Medication relevant to cause of pain.

Annotation or definition.
What is pain?
What causes pain?
How do we classify pain?
What is referred pain?
How do you measure pain?
What are the treatments for pain?

Here are further guidelines.

Analgesics (Pain Relievers)

What are some examples of pain medications?
For what conditions are pain medications used?
What are the differences among the types of pain medications?
What are the side effects of pain medications?
What are the warnings/precautions with pain medications?

Abdominal Pain.
Here are further guidelines.


Back pain
Here are further guidelines.

Chest Pain
Here are further guidelines.

Pain Control after Surgery
Pain Control after Surgery
Pain Disability Questionnaire (PDQ)
Analgesics (Pain Relievers)


Psychogenic Pain
Here are further guidelines.

Pain Resources
Pain Types and Classifications
Vascular Pain
What is pain?
In medicine pain relates to a sensation that hurts. If you feel pain it hurts, you feel discomfort, distress and perhaps agony, depending on the severity of it. Pain can be steady and constant, in which case it may be an ache. It might be a throbbing pain - a pulsating pain. The pain could have a pinching sensation, or a stabbing one.

Types of painAcute pain - this can be intense and short-lived, in which case we call it acute pain. Acute pain may be an indication of an injury. When the injury heals the pain usually goes away.

Chronic pain - this sensation lasts much longer than acute pain. Chronic pain can be mild or intense (severe).

How do we classify pain?
Pain can be nociceptive, non-nociveptive, somatic, visceral, neuropathic, or sympathetic: ¦Nociceptive Pain
- Somatic
- Visceral

¦Non-Nociceptive
- Neuropathic
- Sympathetic
Nociceptive Pain - specific pain receptors are stimulated. These receptors sense temperature (hot/cold), vibration, stretch, and chemicals released from damaged cells.

Somatic Pain - a type of nociceptive pain. Pain felt on the skin, muscle, joints, bones and ligaments is called somatic pain. The term musculo-skeletal pain means somatic pain. The pain receptors are sensitive to temperature (hot/cold), vibration, and stretch (in the muscles). They are also sensitive to inflammation, as would happen if you cut yourself, sprain something that causes tissue damage.

Pain as a result of lack of oxygen, as in ischemic muscle cramps, are a type of nociceptive pain. Somatic pain is generally sharp and well localized - if you touch it or move the affected area the pain will worsen.

Visceral Pain - a type of nociceptive pain. It is felt in the internal organs and main body cavities. The cavities are divided into the thorax (lungs and heart), abdomen (bowels, spleen, liver and kidneys), and the pelvis (ovaries, bladder, and the womb). The pain receptors - nociceptors - sense inflammation, stretch and ischemia (oxygen starvation).

Visceral pain is more difficult to localize than somatic pain. The sensation is more likely to be a vague deep ache. Colicky and cramping sensations are generally types of visceral pain. Visceral pain commonly refers to some type of back pain - pelvic pain generally refers to the lower back, abdominal pain to the mid-back, and thoracic pain to the upper back (see below for the meaning of referred pain).

Nerve Pain or Neuropathic Pain

Nerve pain is also known as neuropathic pain. It is a type of non-nociceptive pain. It comes from within the nervous system itself. People often refer to it as pinched nerve, or trapped nerve. The pain can originate from the nerves between the tissues and the spinal cord (peripheral nervous system) and the nerves between the spinal cord and the brain (central nervous system, or CNS).

Neuropathic pain can be caused by nerve degeneration, as might be the case in a stroke, multiple-sclerosis, or oxygen starvation. It could be due to a trapped nerve, meaning there is pressure on the nerve. A torn or slipped disc will cause nerve inflammation, which will trigger neuropathic pain. Nerve infection, such as shingles, can also cause neuropathic pain.

Neuropathic pain can be caused by nerve degeneration, as might be the case in a stroke, multiple-sclerosis, or oxygen starvation. It could be due to a trapped nerve, meaning there is pressure on the nerve. A torn or slipped disc will cause nerve inflammation, which will trigger neuropathic pain. Nerve infection, such as shingles, can also cause neuropathic pain.

Pain that comes from the nervous system is called non-nociceptive because there are no specific pain receptors. Nociceptive in this text means responding to pain. When a nerve is injured it becomes unstable and its signaling system becomes muddled and haphazard. The brain interprets these abnormal signals as pain. This randomness can also cause other sensations, such as numbness, pins and needles, tingling, and hypersensitivity to temperature, vibration and touch. The pain can sometimes be unpredictable because of this.

Sympathetic Pain

The sympathetic nervous system controls our blood flow to our skin and muscles, perspiration (sweating) by the skin, and how quickly the peripheral nervous system works.

Sympathetic pain occurs generally after a fracture or a soft tissue injury of the limbs. This pain is non-nociceptive - there are no specific pain receptors. As with neuropathic pain, the nerve is injured, becomes unstable and fires off random, chaotic, abnormal signals to the brain, which interprets them as pain.

Generally with this kind of pain the skin and the area around the injury become extremely sensitive. The pain often becomes so intense that the sufferer daren't use the affected arm or leg. Lack of limb use after a time can cause other problems, such as muscle wasting, osteoporosis, and stiffness in the joints.

< If you feel pain it hurts, you feel discomfort, distress and perhaps agony, depending on the severity of it. Pain can be steady and constant, in which case it may be an ache. It might be a throbbing pain - a pulsating pain. The pain could have a pinching sensation, or a stabbing one.

Only the person who is experiencing the pain can describe it properly. Pain is a very individual experience.

Types of pain

Acute pain - this can be intense and short-lived, in which case we call it acute pain. Acute pain may be an indication of an injury. When the injury heals the pain usually goes away.

Chronic pain - this sensation lasts much longer than acute pain. Chronic pain can be mild or intense (severe).

What is referred pain?
Also known as reflective pain. When pain is felt either next to, or at a distance from the origin of an injury it is called referred pain. For example, when a person has a heart attack, even though the affected area is the heart, the pain is sometimes felt around the shoulders, back and neck, rather than in the chest. We have known about referred pain for centuries, but we still do not know its origins and what causes it.

How do you measure pain?
It is virtually impossible to measure a person's pain objectively. Most experts say that the best way to find out how much pain a person is enduring is by a subjective pain report. A comprehensive assessment of pain should include:
The identification of all the pains. This must include the most important ones.
The site, quality, and radiation of pain
What factors aggravate and relieve the pain?
When the pain occurs throughout the day?
What impact the pain has on the person's function?
What impact the pain has on the person's mood?
The sufferers' understanding of their pain
There are many different methods for measuring pain and its severity. Health care professionals say it is important to stick to whatever system or tool you chose for a specific patient all the way through. If a patient is unable to report his pain, such as an infant, or a person with dementia, there are a number of observational pain measures a doctor can use.

Here is a list of some pain measures used today:

Numerical Rating Scales

The patient is given a form which asks him to tick from 0 to 10 what his level of pain is. 0 is no pain, 5 is moderate pain, and 10 is the worst pain imaginable.

Please rate the pain you have right now
0 2 3 4 5 6 7 8 9 10

No pain Moderate pain Worst pain imaginable

The Numerical Rating Scales are useful if you want to measure any changes in pain, as well as gauging the patient's response to pain treatment. If the patient has dyslexia, autism, or is very elderly and has dementia this may not be the best tool (see the ones below).

Verbal Descriptor Scale

This type of scale exists in many different forms. The patient is asked questions and responds verbally choosing from such terms as mild, moderate, severe, no pain, mild pain, discomforting, distressing, horrible, and excruciating.

Elderly patients with cognitive impairment, very young children, and people who respond better to verbal stimuli tend to have better completion rates with this type of scale, compared to the written numerical scale. Children respond even better to the faces scale (description below).

Faces Scale

The patient sees a series of faces. The first one is calm and happy, the second less so, etc., and the final one has an expression of extreme pain. This scale is used mainly for children, but can also be used with elderly patients with cognitive impairment. Patients with autism may respond better to this type of approach - people with autism tend to respond to visual stimuli well.

Brief Pain Inventory

This is a much more comprehensive written questionnaire. Not only does it gauge current level of pain, but also records the peaks and troughs of pain during previous days, how pain has affected mood, activity, sleep patterns, and how the pain may have affected the patient's interpersonal relationship. The questionnaire also has diagrams which the patient shades - the shaded parts being where the pain is located and where it is most severe.

Pain Questionnaire

This questionnaire measures the intensity (severity) of the pain, the quality of the pain, mood, and understanding of the pain.

Look at the 20 groups below.
1.Circle one word in each group that best describes your pain.
2.Circle only three words from Groups 1 to 10 that best describe your pain response.
3.Choose just two words in Groups 11 to 15 that best describe your pain.
4.Just pick the one in Group 16.
5.Finally, choose just one word from Groups 17-20.
You should now have seven words. Those seven words should be taken to your doctor. They will help describe both the quality and intensity of your pain.

Group 1 - Flickering, Pulsing, Quivering, Throbbing, Beating, Pounding
Group 2 - Jumping, Flashing, Shooting
Group 3 - Pricking, Boring, Drilling, Stabbing
Group 4 - Sharp, Gritting, Lacerating
Group 5 - Pinching, Pressing, Gnawing, Cramping, Crushing
Group 6 - Tugging, Pulling, Wrenching
Group 7 - Hot, Burning, Scalding, Searing
Group 8 - Tingling, Itching, Smarting, Stinging
Group 9 - Dull, Sore, Hurting, Aching, Heavy
Group 10 - Tender, Taunt, Rasping, Splitting
Group 11 - Tiring, Exhausting
Group 12 - Sickening, Suffocating
Group 13 - Fearful, Frightful, Terrifying
Group 14 - Punishing, Grueling, Cruel, Vicious, Killing
Group 15 - Wretched, Binding
Group 16 - Annoying, Troublesome, Miserable, Intense, Unbearable
Group 17 - Spreading, Radiating, Penetrating, Piercing
Group 18 - Tight, Numb, Squeezing, Drawing, Tearing
Group 19 - Cool, Cold, Freezing
Group 20 - Nagging, Nauseating, Agonizing, Dreadful, Torturing

Measuring pain when the patient is cognitively impaired

In this case doctors say that the patient's subjective pain report is the most effective and accurate way of evaluating pain. If the severely cognitively impaired patient is observed carefully it is possible to pick out clues as to the presence of pain, e.g. restlessness, crying, moaning, groaning, grimacing, resistance to care, reduced social interactions, increased wandering, not eating, and sleeping problems.

Chronic pain may be genetic, new research suggests - researchers asked 2,721 patients with chronic pain to rate the intensity of their pain from 0 to 10. All of the patients were taking prescribed opioid pain medications. The research team found that a gene variant, DRD1, was 33% more common in the low pain group than in the high pain group. The people in the moderate pain group were more likely to have another two variants - COMT, which was 25% more common in this group than in the high pain group, and OPRK, which was 19% more common. The high pain group, meanwhile, were 25% more likely to have the variant DRD2 than the people in the moderate group.

Chronic pain linked to vitamin D deficiency in men - vitamin D deficiency has been linked to a number of health issues and a new study suggests that low levels of vitamin D in the body are linked to chronic widespread pain. Results show that those with vitamin D deficiency at the start of the study were more than twice as likely to experience chronic widespread pain, compared with those who had the highest levels.

What are the treatments for pain?An underlying disorder, if treated effectively, will also get rid of the pain, or at least reduce it. If you have an infection and take antibiotics, the antibiotics may get rid of that infection, resulting also in the elimination of pain. Even if an underlying problem can be treated, you may still need analgesics (pain relievers).

Analgesics are good at relieving nociceptive pain, but not neuropathic pain. Chronic pain - long-lasting pain - may need other non-drug treatments as well.

Opioid Analgesics

Opioid analgesics are also known as narcotics. These are the strongest painkillers and are commonly used after surgery, for cancer, broken bones, burns, and various other situations. Even though opioids are not commonly used to treat non-cancer pain, their usage for non-cancer pain is becoming more widespread and acceptable. Some patients do not respond well to opioids and should not take them.

The patient will be given opioids in gradually increasing dosages. The ideal dose is reached when the pain is relieved and the side-effects are tolerable (increase any higher and the side effects become too much for the patient). Dosages should be generally much lower for older patients and infants.

The patient is administered opioids every few hours - each dose coinciding with the moment just before the pain starts becoming severe. Some patients are given higher dosages if the pain becomes more intense, while others are given other medications alongside the opioid. Pain can become more intense if the patient needs to move about, or if a wound dressing needs to be changed.

The dosage goes down if the pain intensity drops, until if possible, the doctor switches to a non-opioid analgesic.

People with kidney failure, liver problems, COPD (chronic obstructive pulmonary disease, dementia, tend to have more side effects when given opioids. The most common opioid side effects are drowsiness, constipation, nausea, vomiting, and itching. Generally, the side effects lessen as after time. Taking too much opioid can be dangerous. Patients who take opioids for long period become physically dependent and will have withdrawal symptoms when treatment is stopped - it is important that their dosage is tapered off gradually.

Codeine may increase pain sensitivity if used frequently - researchers at The University of Adelaide in Australia reported on a study that found that frequent and large doses of codeine can result in greater sensitivity to pain.

Nonopioid Analgesics

Nonopioid analgesics are used generally for mild to moderate pain. They are not addictive and their pain-relieving effects do not dwindle over time.

NSAIDs (nonsteroidal anti-inflammatory drugs)

These may be obtained either OTC (over-the-counter) or as a prescription medication, it depends on the dosage. Low dosage NSAIDs are effective for headaches, muscle aches, fever, and minor pains. At a higher dose they help reduce joint inflammation. There are three main types of NSAIDs, and they all block prostaglandins - hormone-like substances that cause pain, inflammation, muscle cramps, and fever.

¦Traditional NSAIDs - the largest subset of NSAIDs. As is the case with most drugs, they do carry a risk of side-effects, such as stomach upset and gastrointestinal bleeding. The risk of side effects is significantly higher if the patient is over 60. At higher doses, they should only be taken when monitored by a doctor.

¦COX-2 inhibitors - these also reduce pain and inflammation. However, they are designed to have fewer stomach and gastrointestinal side-effects. In 22004/2005 Vioxx and Bextra were withdrawn from the market after major studies showed Vioxx carried increased cardiovascular risks, while Bextra triggered serious skin reactions. Some other COX-2 inhibitors are also being investigated for side-effects. The FDA told makers of NSAIDs to highlight warnings on their labels in a black box.

¦Salicylates - these include aspirin which continues to be a popular medication for many doctors and patients. If your plan to take aspirin more than just occasionally you should consult your doctor. Long term high dosage usage of aspirin carries with it a significant risk of serious undesirable side effects, such as kidney problems and gastrointestinal bleeding. For effective control of arthritis pain and inflammation frequent large doses are needed.

Nonacetylated salicylate is designed to have fewer side effects than aspirin. Some doctors may prescribe nonacetylated salicylate if they feel aspirin is too risky for their patient. Nonacetylated salicylate does not have the chemical aspirin has which protects against cardiovascular disease. Some doctors prescribe low dose aspirin along with nonacetylated salicylate for patients who they feel need cardiovascular protection.

History relevant to pain.
Pain Assessment Scales
Rating scale is recommended for persons age 3 years and older.

Do You Have Pain?
Where is the Pain?
Is there more than one site?
When did your pain start?
How often does it occur?
Has its intensity changed?
How long does it last?
What makes your pain worse?
What makes your pain better?
What previous treatment have you tried to relieve your pain?
Were they effective?
What does your pain feel like?
What words would you use to describe your pain?
Does the pain move anywhere?
On a scale of 0 to 10 with O being no pain and 10 being the worst pain you can imagine, how much does it hurt right now?
How much does it hurt at it’s worst?
How much does it hurt at it’s best?
If your medicine helps with the pain, how many hours of relief do you get?


Pain History and Pain Assessment
Managing Pain After Surgery

Pain History and Pain Assessment

Where does it hurt?
When did it start?
Does it go anywhere else?
Where is your pain? Is there more than one site?
What makes your pain worse?
What makes your pain better?
What previous treatment have you tried to relieve your pain?
Were they effective?
What does your pain feel like?
What words would you use to describe your pain?
Does the pain move anywhere?
On a scale of 0 to 10 with O being no pain and 10 being the worst pain you can imagine, how much does it hurt right now?
How much does it hurt at it’s worst?
How much does it hurt at it’s best?
When did your pain start?
How often does it occur?
Has its intensity changed?
How long does it last?


Aggravating/ Alleviating
Associated symptoms
Alleviating factors
Aggravating factors
Associated factors

Before (ever happened before)

Character (stabbing, throbbing, etc.)

Duration

Exacerbating factors

Frequency

Intensity

Irritating and relieving factors

Location

Onset of pain (time, duration)

Precipitating factors

Palliative factors for pain

Quality of pain (throbbing, stabbing, dull, etc.)

Region of body affected

Radiation (how does pain spread)

Relieving factors

Severity of pain (usually scale of 1-10)

Symptoms associated
Setting

Site (Main site)

Timing of pain (duration, frequency, after exercise, in evening, etc.)
U: How does it affect 'U' in your daily life?
Provocative factors
Treatments tried
Deja Vu: Has this happened before?
Worry: What do you think or fear that it is?
Worsening factors

Site

Where does it hurt?

Onset

When did it start?

Sudden (seconds to minutes)?
Gradual (hours to days )?
Character
Heavy, burning, stabbing, aching? Constant, intermittent, related to time of day/ activity?
Like anything else you have felt before?

Radiation

Does it go anywhere else?
Associated symptoms
Depends on type of pain/ discomfort

Timing

Exact sequence of events leading to onset of pain
The time course of the pain after onset
Exacerbating/ Relieving Factors
Factors that the patients have noted make the pain worse/ better
Exercise, movement, breathing, analgesia, position Severity
You should ask patients to quantify their pain although this is very subjective
On a scale of 1 to 10 if 10 is the worst pain ever how would you rate your pain

Assessment of Pain: Questions to Consider during Assessment of Pain (PQRST)

Identify Location of Pain

Ask: Where is your pain? Is there more than one site?

Identify PQRST characteristics:

P - provocating and precipitating factors, relieving factors

Ask:

What makes your pain worse?
What makes your pain better?
What previous treatment have you tried to relieve your pain?
Were they effective?

Q - quality of pain (eg. burning, stabbing, gnawing, shooting, lancinating)

Ask:

What does your pain feel like?
What words would you use to describe your pain?
R - radiation

Ask:

Does the pain move anywhere?
S - severity (use an appropriate intensity scale - see Appendix E)

Ask:

On a scale of 0 to 10 with O being no pain and 10 being the worst pain you can imagine, how much does it hurt right now?
How much does it hurt at it’s worst?
How much does it hurt at it’s best?
T - timing

Ask:

When did your pain start?
How often does it occur?
Has its intensity changed?
How long does it last?
The Assessment of the Patient with Pain
Question to Ask Yourself/Questions to Ask Your Doctor
Classification of Pain

Defining Pain
Classifying Pain
Types of Pain
What Is Pain?
Types of pain
How do we classify pain?
What Causes Pain?
What is referred pain?
How do you measure pain?
What are the treatments for pain?

In medicine pain relates to a sensation that hurts. If you feel pain it hurts, you feel discomfort, distress and perhaps agony, depending on the severity of it. Pain can be steady and constant, in which case it may be an ache. It might be a throbbing pain - a pulsating pain. The pain could have a pinching sensation, or a stabbing one.

Only the person who is experiencing the pain can describe it properly. Pain is a very individual experience.

According to _________ medical dictionary, Pain is:

1. A variably unpleasant sensation associated with actual or potential tissue damage and mediated by specific nerve fibers to the brain where its conscious appreciation may be modified by various factors.

2. Term used to denote a painful uterine contraction occurring in childbirth.

Types of pain

Acute pain - this can be intense and short-lived, in which case we call it acute pain. Acute pain may be an indication of an injury. When the injury heals the pain usually goes away.

Chronic pain - this sensation lasts much longer than acute pain. Chronic pain can be mild or intense (severe).

How do we classify pain?

Pain can be nociceptive, non-nociveptive, somatic, visceral, neuropathic, or sympathetic. Look at the table below.

Pain

Nociceptive Non-Nociceptive

Somatic Visceral Neuropathic Sympathetic

Nociceptive Pain - specific pain receptors are stimulated. These receptors sense temperature (hot/cold), vibration, stretch, and chemicals released from damaged cells.

Somatic Pain - a type of nociceptive pain. Pain felt on the skin, muscle, joints, bones and ligaments is called somatic pain. The term musculo-skeletal pain means somatic pain. The pain receptors are sensitive to temperature (hot/cold), vibration, and stretch (in the muscles). They are also sensitive to inflammation, as would happen if you cut yourself, sprain something that causes tissue damage. Pain as a result of lack of oxygen, as in ischemic muscle cramps, are a type of nociceptive pain. Somatic pain is generally sharp and well localized - if you touch it or move the affected area the pain will worsen.

Visceral Pain - a type of nociceptive pain. It is felt in the internal organs and main body cavities. The cavities are divided into the thorax (lungs and heart), abdomen (bowels, spleen, liver and kidneys), and the pelvis (ovaries, bladder, and the womb). The pain receptors - nociceptors - sense inflammation, stretch and ischemia (oxygen starvation).

Visceral pain is more difficult to localize than somatic pain. The sensation is more likely to be a vague deep ache. Colicky and cramping sensations are generally types of visceral pain. Visceral pain commonly refers to some type of back pain - pelvic pain generally refers to the lower back, abdominal pain to the mid-back, and thoracic pain to the upper back (see below for the meaning of referred pain).

Nerve Pain or Neuropathic Pain

Nerve pain is also known as neuropathic pain. It is a type of non-nociceptive pain. It comes from within the nervous system itself. People often refer to it as pinched nerve, or trapped nerve. The pain can originate from the nerves between the tissues and the spinal cord (peripheral nervous system) and the nerves between the spinal cord and the brain (central nervous system, or CNS).

Neuropathic pain can be caused by nerve degeneration, as might be the case in a stroke, multiple-sclerosis, or oxygen starvation. It could be due to a trapped nerve, meaning there is pressure on the nerve. A torn or slipped disc will cause nerve inflammation, which will trigger neuropathic pain. Nerve infection, such as shingles, can also cause neuropathic pain.

Pain that comes from the nervous system is called non-nociceptive because there are no specific pain receptors. Nociceptive in this text means responding to pain. When a nerve is injured it becomes unstable and its signaling system becomes muddled and haphazard. The brain interprets these abnormal signals as pain. This randomness can also cause other sensations, such as numbness, pins and needles, tingling, and hypersensitivity to temperature, vibration and touch. The pain can sometimes be unpredictable because of this.

Sympathetic Pain

The sympathetic nervous system controls our blood flow to our skin and muscles, perspiration (sweating) by the skin, and how quickly the peripheral nervous system works.

Sympathetic pain occurs generally after a fracture or a soft tissue injury of the limbs. This pain is non-nociceptive - there are no specific pain receptors. As with neuropathic pain, the nerve is injured, becomes unstable and fires off random, chaotic, abnormal signals to the brain, which interprets them as pain.

Generally with this kind of pain the skin and the area around the injury become extremely sensitive. The pain often becomes so intense that the sufferer daren't use the affected arm or leg. Lack of limb use after a time can cause other problems, such as muscle wasting, osteoporosis, and stiffness in the joints.

Acute pain can last a moment; rarely does it become chronic pain. Chronic pain persists for long periods. It is resistant to most medical treatments and cause severe problems.

What Is Pain?



Pain Basics

Pain attacks the human body at every vulnerable target -- muscles, bones, and joints. Click here for an overview -- causes and treatments of acute and chronic pain.

Chronic Pain

Learn about how chronic pain occurs, and why chronic pain sometimes lingers.

Causes

What Causes Chronic Pain?

Anything from a bad mattress to stomach ulcers can cause chronic pain. While it may begin with an injury or illness, pain can develop a psychological dimension once the physical problem heals.

Nerve Pain

When nerve fibers get damaged, the result can be chronic pain. Read about the very common causes of neuropathic pain, like diabetes.

What Causes Chest Pain?

Certainly chest pain is not something to ignore. But you should know that it has many possible causes.

Burns and Pain

Burns differ in type and severity -- as does the pain.

Compressed Nerve (Pinched Nerve)

Pinched nerves can sometimes lead to other conditions such as peripheral neuropathy, carpal tunnel syndrome, and tennis elbow. Early diagnosis can prevent damage read more.

Foot Pain

Foot pain is one of the most common problems people experience.

Hand Pain Causes

Hand pain has many causes, including injury and disease. Fortunately, many of those causes can be treated and the symptoms eased.

Knee Pain

With today’s increasingly active society, the number of knee problems is increasing. Knee pain has a wide variety of specific causes and treatments.

Lower Leg Pain

If you're suffering from lower leg pain, you may wonder if it's serious or something you can treat at home.

Elbow Pain

Find out about the common causes and treatments of elbow pain.

Slideshow: Low Back Pain

Fortunately, low back pain often gets better on its own. When it doesn't, there are effective treatments.

What is referred pain?

Also known as reflective pain. When pain is felt either next to, or at a distance from the origin of an injury it is called referred pain. For example, when a person has a heart attack, even though the affected area is the heart, the pain is sometimes felt around the shoulders, back and neck, rather than in the chest. We have known about referred pain for centuries, but we still do not know its origins and what causes it.

How do you measure pain?

It is virtually impossible to measure a person's pain objectively. Most experts say that the best way to find out how much pain a person is enduring is by a subjective pain report. A comprehensive assessment of pain should include:

The identification of all the pains. This must include the most important ones.

The site, quality, and radiation of pain

What factors aggravate and relieve the pain?

When the pain occurs throughout the day?

What impact the pain has on the person's function?

What impact the pain has on the person's mood?

The sufferers' understanding of their pain There are many different methods for measuring pain and its severity. Health care professionals say it is important to stick to whatever system or tool you chose for a specific patient all the way through. If a patient is unable to report his pain, such as an infant, or a person with dementia, there are a number of observational pain measures a doctor can use.

Here is a list of some pain measures used today:

Numerical Rating Scales

The patient is given a form which asks him to tick from 0 to 10 what his level of pain is. 0 is no pain, 5 is moderate pain, and 10 is the worst pain imaginable.

Please rate the pain you have right now

0 2 3 4 5 6 7 8 9 10

No pain Moderate pain Worst pain imaginable

The Numerical Rating Scales are useful if you want to measure any changes in pain, as well as gauging the patient's response to pain treatment. If the patient has dyslexia, autism, or is very elderly and has dementia this may not be the best tool (see the ones below).

Verbal Descriptor Scale

This type of scale exists in many different forms. The patient is asked questions and responds verbally choosing from such terms as mild, moderate, severe, no pain, mild pain, discomforting, distressing, horrible, and excruciating.

Elderly patients with cognitive impairment, very young children, and people who respond better to verbal stimuli tend to have better completion rates with this type of scale, compared to the written numerical scale. Children respond even better to the faces scale (description below).

Faces Scale

The patient sees a series of faces. The first one is calm and happy, the second less so, etc., and the final one has an expression of extreme pain. This scale is used mainly for children, but can also be used with elderly patients with cognitive impairment. Patients with autism may respond better to this type of approach - people with autism tend to respond to visual stimuli well.

Brief Pain Inventory

This is a much more comprehensive written questionnaire. Not only does it gauge current level of pain, but also records the peaks and troughs of pain during previous days, how pain has affected mood, activity, sleep patterns, and how the pain may have affected the patient's interpersonal relationship. The questionnaire also has diagrams which the patient shades - the shaded parts being where the pain is located and where it is most severe.

McGill Pain Questionnaire

This questionnaire measures the intensity (severity) of the pain, the quality of the pain, mood, and understanding of the pain. It is also known as the McGill Pain Index. It is a scale of rating pain developed at McGill University by Melzack and Torgerson (1971).

Look at the 20 groups below.

Circle one word in each group that best describes your pain.

Circle only three words from Groups 1 to 10 that best describe your pain response.

Choose just two words in Groups 11 to 15 that best describe your pain.

Just pick the one in Group 16.

Finally, choose just one word from Groups 17-20. You should now have seven words. Those seven words should be taken to your doctor. They will help describe both the quality and intensity of your pain.

Group 1 - Flickering, Pulsing, Quivering, Throbbing, Beating, Pounding

Group 2 - Jumping, Flashing, Shooting

Group 3 - Pricking, Boring, Drilling, Stabbing

Group 4 - Sharp, Gritting, Lacerating

Group 5 - Pinching, Pressing, Gnawing, Cramping, Crushing

Group 6 - Tugging, Pulling, Wrenching

Group 7 - Hot, Burning, Scalding, Searing

Group 8 - Tingling, Itching, Smarting, Stinging

Group 9 - Dull, Sore, Hurting, Aching, Heavy

Group 10 - Tender, Taunt, Rasping, Splitting

Group 11 - Tiring, Exhausting

Group 12 - Sickening, Suffocating

Group 13 - Fearful, Frightful, Terrifying

Group 14 - Punishing, Grueling, Cruel, Vicious, Killing

Group 15 - Wretched, Binding

Group 16 - Annoying, Troublesome, Miserable, Intense, Unbearable

Group 17 - Spreading, Radiating, Penetrating, Piercing

Group 18 - Tight, Numb, Squeezing, Drawing, Tearing

Group 19 - Cool, Cold, Freezing

Group 20 - Nagging, Nauseating, Agonizing, Dreadful, Torturing

Measuring pain when the patient is cognitively impaired

In this case doctors say that the patient's subjective pain report is the most effective and accurate way of evaluating pain. If the severely cognitively impaired patient is observed carefully it is possible to pick out clues as to the presence of pain, e.g. restlessness, crying, moaning, groaning, grimacing, resistance to care, reduced social interactions, increased wandering, not eating, and sleeping problems.

What are the treatments for pain?

An underlying disorder, if treated effectively, will also get rid of the pain, or at least reduce it. If you have an infection and take antibiotics, the antibiotics may get rid of that infection, resulting also in the elimination of pain. Even if an underlying problem can be treated, you may still need analgesics (pain relievers).

Analgesics are good at relieving nociceptive pain, but not neuropathic pain. Chronic pain - long-lasting pain - may need other non-drug treatments as well.

Opioid Analgesics

Opioid analgesics are also known as narcotics. These are the strongest painkillers and are commonly used after surgery, for cancer, broken bones, burns, and various other situations. Even though opioids are not commonly used to treat non-cancer pain, their usage for non-cancer pain is becoming more widespread and acceptable. Some patients do not respond well to opioids and should not take them.

The patient will be given opioids in gradually increasing dosages. The ideal dose is reached when the pain is relieved and the side-effects are tolerable (increase any higher and the side effects become too much for the patient). Dosages should be generally much lower for older patients and infants.

The patient is administered opioids every few hours - each dose coinciding with the moment just before the pain starts becoming severe. Some patients are given higher dosages if the pain becomes more intense, while others are given other medications alongside the opioid. Pain can become more intense if the patient needs to move about, or if a wound dressing needs to be changed.

The dosage goes down if the pain intensity drops, until if possible, the doctor switches to a non-opioid analgesic.

People with kidney failure, liver problems, COPD (chronic obstructive pulmonary disease, dementia, tend to have more side effects when given opioids. The most common opioid side effects are drowsiness, constipation, nausea, vomiting, and itching. Generally, the side effects lessen as after time. Taking too much opioid can be dangerous. Patients who take opioids for long period become physically dependent and will have withdrawal symptoms when treatment is stopped - it is important that their dosage is tapered off gradually.

Nonopioid Analgesics

Nonopioid analgesics are used generally for mild to moderate pain. They are not addictive and their pain-relieving effects do not dwindle over time.

NSAIDs (nonsteroidal anti-inflammatory drugs)

These may be obtained either OTC (over-the-counter) or as a prescription medication, it depends on the dosage. Low dosage NSAIDs are effective for headaches, muscle aches, fever, and minor pains. At a higher dose they help reduce joint inflammation. There are three main types of NSAIDs, and they all block prostaglandins - hormone-like substances that cause pain, inflammation, muscle cramps, and fever. ??

Traditional NSAIDs - the largest subset of NSAIDs. As is the case with most drugs, they do carry a risk of side-effects, such as stomach upset and gastrointestinal bleeding. The risk of side effects is significantly higher if the patient is over 60. At higher doses, they should only be taken when monitored by a doctor.

COX-2 inhibitors - these also reduce pain and inflammation. However, they are designed to have fewer stomach and gastrointestinal side-effects. In 22004/2005 Vioxx and Bextra were withdrawn from the market after major studies showed Vioxx carried increased cardiovascular risks, while Bextra triggered serious skin reactions. Some other COX-2 inhibitors are also being investigated for side-effects. The FDA told makers of NSAIDs to highlight warnings on their labels in a black box.

Salicylates - these include aspirin which continues to be a popular medication for many doctors and patients. If your plan to take aspirin more than just occasionally you should consult your doctor. Long term high dosage usage of aspirin carries with it a significant risk of serious undesirable side effects, such as kidney problems and gastrointestinal bleeding. For effective control of arthritis pain and inflammation frequent large doses are needed. Nonacetylated salicylate is designed to have fewer side effects than aspirin. Some doctors may prescribe nonacetylated salicylate if they feel aspirin is too risky for their patient. Nonacetylated salicylate does not have the chemical aspirin has which protects against cardiovascular disease. Some doctors prescribe low dose aspirin along with nonacetylated salicylate for patients who they feel need cardiovascular protection.

The Assessment of the Patient with Pain
Core questions to be answered as part of a pain assessment:
Pain assessment in the elderly
Pain assessment in infants
Pain assessment in young children
1. Introduction
2. Question to Ask Yourself
3. Questions to Ask Your Doctor
4. Further Information
Acute vs. Chronic Pain
Psychogenic Pain
Pain Control after Surgery
Abdominal Pain
Vascular Pain
Pain Resources
Treatment & Care

Pain Pathway

Anticipate what the doctor needs to know

Let's say that, for example, you have pain. (If you have more than one type of pain you may need to describe each pain separately.) You should be prepared to answer the following questions:

Where is the pain most severe?
When did it start?
Does anything trigger it?
Is there anything you could do to bring it on, make it better, make it worse?
Is it present every day, or do you have pain free days?
Is it worst in the morning, as the day goes on, or constant?
On a scale of 1-10, how severe is the pain?
Is it constant or off and on?
Do you have any other symptoms with it, such as chest pain, shortness of breath?
Does the pain stay in one area or spread to other areas?
Does it interfere with your daily routine?
What has been its course? (Is it stable, or getting worse?)
Is this a new symptom or a recurrence of a previous problem?
Back Pain
How do we classify pain?
What is referred pain?
How do you measure pain?
What are the treatments for pain?
What are the basic types of pain?
What is Pain?

Pain is a feeling triggered in the nervous system. Pain may be sharp or dull. It may come and go, or it may be constant. You may feel pain in one area of your body, such as your back, abdomen or chest or you may feel pain all over, such as when your muscles ache from the flu.

Pain can be helpful in diagnosing a problem. Without pain, you might seriously hurt yourself without knowing it, or you might not realize you have a medical problem that needs treatment. Once you take care of the problem, pain usually goes away. However, sometimes pain goes on for weeks, months or even years. This is called chronic pain. Sometimes chronic pain is due to an ongoing cause, such as cancer or arthritis. Sometimes the cause is unknown.

Fortunately, there are many ways to treat pain. Treatment varies depending on the cause of pain. Pain relievers, acupuncture and sometimes surgery are helpful.

Defining Pain

Pain is a perception that signals the individual that tissue damage has occurred or may be occurring. It is subjective and very complex. The processes in the body that are involved in the perception of pain are called "nociception." Basic and clinical research during the past 50 years has confirmed that there are many mechanisms involved in nociception.

Classifying Pain

Pain can be "acute" or "chronic." Acute pain lasts a short time, or is expected to be over soon. The time frame may be as brief as seconds or as long as weeks.

Chronic pain may be defined as pain that lasts beyond the healing of an injury, continues for a period of several months or longer, or occurs frequently for at least months.

To develop the best treatment strategies, health care professionals also classify pain based on its characteristics, its cause, or the mechanisms in the body or the mind that are probably involved in sustaining it. One common classification based on mechanisms distinguishes pain into categories called "nociceptive," "neuropathic," and "psychogenic."

Types of Pain

Nociceptive Pain: Nociceptive pain is believed to be caused by the ongoing activation of pain receptors in either the surface or deep tissues of the body. There are two types: "somatic" pain and " visceral" pain.

"Somatic" pain is caused by injury to skin, muscles, bone, joint, and connective tissues. Deep somatic pain is usually described as dull or aching, and localized in one area. Somatic pain from injury to the skin or the tissues just below it often is sharper and may have a burning or pricking quality.

Somatic pain often involves inflammation of injured tissue. Although inflammation is a normal response of the body to injury, and is essential for healing, inflammation that does not disappear with time can result in a chronically painful disease. The joint pain caused by rheumatoid arthritis may be considered an example of this type of somatic nociceptive pain.

"Visceral" pain refers to pain that originates from ongoing injury to the internal organs or the tissues that support them. When the injured tissue is a hollow structure, like the intestine or the gall bladder, the pain often is poorly localized and cramping. When the injured structure is not a hollow organ, the pain may be pressure-like, deep, and stabbing.

Neuropathic Pain: Neuropathic pain is believed to be caused by changes in the nervous system that sustain pain even after an injury heals. In most cases, the injury that starts the pain involves the peripheral nerves or the central nervous system itself. It can be associated with trauma or with many different types of diseases, such as diabetes. There are many neuropathic pain syndromes, such as diabetic neuropathy, trigeminal neuralgia, postherpetic neuralgia ("shingles"), post-stroke pain, and complex regional pain syndromes (also called reflex sympathetic dystrophy or "RSD" and causalgia). Some patients who get neuropathic pain describe it as bizarre, unfamiliar pain, which may be burning or like electricity. The pain may be associated with sensitivity of the skin.

Psychogenic Pain: Most patients with chronic pain have some degree of psychological disturbance. Patients may be anxious or depressed, or have trouble coping. Psychological distress may not only be a consequence of the pain, but may also contribute to the pain itself. "Psychogenic" pain is a simple label for all kinds of pain that can be best explained by psychological problems.

This close relationship between pain and psychological distress means that all patients with chronic pain should have an assessment of these psychological factors, and psychological treatments should be considered an important aspect of pain therapy. In some cases, psychological problems appear to be a main cause of the pain. This does not mean that the person is not actually experiencing the pain. Rather, the patient is truly suffering but the main cause somehow relates to the emotions, or to learning, or to some other psychological process. Although doctors sometimes encounter patients who pretend to be in pain (some can be called malingerers), this appears to be a rare occurrence. Most patients with pain that appears to be determined primarily by psychological processes are hurting just like those who have pain associated with a clear injury to the body.

Sometimes, psychogenic pain occurs in the absence of any identifiable disease in the body. More often, there is a physical problem but the psychological cause for the pain is believed to be the major cause for the pain. The various types of psychogenic pain can be diagnosed using a classification developed by psychiatrists (the Diagnostic and Statistical Manual of the American Psychiatric Association).

The following organizations can provide more information about pain:

American Chronic Pain Association PO Box 850 Rocklin, CA 95677 916-632-0922 www.theacpa.org

American Pain Society 4700 W. Lake Avenue Glenview, IL 60025 847-375-4715 www.ampainsoc.org

Q: Who has used the Medisana Back Pain Relief System and why?

I’m thinking about buying this for my lower back. Is this like the shock therapy they use in sports medicine(you know, the one that makes your muscles twitch)?? Sometimes my sciatic nerve sack acts up from being pinched. Will this help me?

A: Yes I use this at physical therapy for my sciatica nerve issues…

Actually woot.com has this on sale today.

Q: Which is the best back pain relief spray/ointment?

I am suffering from a severe lower back pain for the past 2 years… I am working in the IT industry… which demands long sitting for hours… I tried both English and Ayurvedic Medicines… There is no effect at all…

Can anyone please prescribe any pain relief spray/ointment which I can use to avoid the pain temporarily?

A: You could try icy hot. That feels really good, but the side effect is that it stinks really bad. Well, it stinks to me. You can’t prescribe any medicine over the internet. You’ll need to go see a real doctor.

Q: are there complaints against back2life back pain relief machine, does it work?

does this machine work to relieve lower back pain?

A: I just recently saw the infomercial, and also wondered about it, so I’ve started doing some research. A good place to start is

So far, only one complaint, but it’s a big one.

Q: How can I finally get relief from back pain?

I had two spinal operations in recent years and I still have significant pain in my right leg from the original injury. The operations stopped it from getting worse, but didn’t really make it better. I’ve taken all kinds of medications, had electrical stimulation, acupuncture, the works. Any hints?

A: Years ago, one of the world’s leading spine surgeons made this statement at a large medical conference in Las Vegas, Nevada:

“Your first back surgery is your best chance, your second back surgery is your last chance.â€

You are done with surgery. It hasn’t solved your problem (in fact, it may be the cause of your continuing problems).

Couple of suggestions. First, how many Chiropractic evaluations/consults have you had? A good Chiropractor may be a very valuable resource for you.

Second, there is a wonderful book available that explains the many unappreciated causes of chronic pain and how to effectively treat the causes to relieve the pain. If you read this book, you will understand the many ways that surgery and even pain medications can actually aggravate and cause long-term chronic pain.

The book is written by Sherry Rogers, M.D. and is called: “Pain Free in 6 Weeksâ€. It is available at Amazon.com or from the publisher at 1-800-846-6687. I highly recommend this book.

Best wishes and good luck.

Q: Preventing A Sore, Stiff Back – What Works For Back Pain Relief?

Looking for sites on back pain relief exercises as well.

A: Keep your back as straight as possible. Good posture is key to preventing a sore, stiff back. Massage is fantastic to relieve back pain — use some natural oil for greater effect. Regular exercise helps loosen the muscles & joints and maintain body flexibility. This is important to prevent sudden cramps and lower back pain.

Q: What is the best “around the house†relief from back pain.? I do not take ANY types of medications, so I am loking for a temporary “home remedy†for back pain. Thanks!

A: the couch with ur feet up

Q: What’s the best way to get rid of lower right back pain?

I’m looking for a quick way to get rid of lower right back pain without going to a chiropractor or without any meds.

A: Sorry to hear about your pain. I was researching about the same thing, and came across some info on this site. www.lowerrightback-pain.com .It talks about some quick and simple exercises you can do to get rid of the pain. Good luck!

Q: what would be the cause of lower right back pain?

I have lower back pain on my right side and it is a sharp pain. It hurts when I lay on my side and it hurts when sit up or move around. Could anyone please help me with my problem and tell me what’s wrong with me? I’m a broke college student and I don’t have the money to pay for an emergency room bill. Thanks!

A: Hi Rettajean. As you can see, the answers on this site can be quite ‘diverse’.

A correct diagnosis cannot be made from the limited information you give (and without a good physical examiniation). Did your pain start suddenly or gradually? Does anything (e.g. any position) make it better? Have you had this type of pain before? These are some of the basic questions your doctor should ask.

Additional questions include the following. Did you injure yourself or do any excessive activity prior to the onset of the pain (sudden twisting or fall or lifting or sports excesses, ect.)? Are you having any difficutly with urination? Change in bowel function? Menstrual problems? Do you have a fever? Chills or night sweats? Leg pains or numbness? Any muscle weakness in the legs? Does coughing or sneezing aggravate the pain? These are just some of the questions that need to be answered to help guide your healthcare professional to a correct diagnosis.

Note: If you answer ‘yes’ to any of the above questions, you really should see your doctor or Chiropractor.

Have you tried ice (cryotherapy) to see if that will help the pain? If you do not have an icepack designed for this, you can buy one at any drug store or you can use a large bag of frozen vegetables (corn or peas work well).

Place the icepack or veggies in a thin pillow case cover and place that on your area of pain. Apply the icepack for 20 minutes (if you are doing it correctly you will feel periods of cold, burning, aching, pins/needles and then analgesia/numbness). Do not keep the icepack on more than about 20 minutes. You need to give the area a 1 hour period without ice to allow for return of healing blood flow and to prevent frost bite (freezer burn!). After one hour you can apply the icepack again for another 20 minutes. Repeat as needed.

Best wishes and good luck!

Q: is lower right back pain a symptom of pregnancy?

i think this is my month. i am ttc for sometime. i had lower right abdominal cramps for about 3 days….i think that was implantation cramps and today i have mild lower right back pain. has anyone had it before? i going to take pregnancy test on friday as that would be apt. Thanks in advance.

A: I’m 19 weeks along now and when I do have back pain in this pregnancy, it’s always been on the lower right side, for some reason. Not sure if that makes it a sign but it’s possible, as I know I’m definitely pregnant!

Good luck and lots of baby dust!!

Q: Prolonged and Persistent Upper Right Back Pain?

I’ve been having this weird pain (it’s a nagging pain which never really go off at all) at my upper right back for more than a year. The pain is specifically at one spot but it has now spread to the bottom right back (near the lower right rib cage).

Sometimes, when the pain is getting a little too much to bear, it’ll be painful to breathe in deep.

Anyone encountered this problem before? What is your diagnosis?

A: Been there. Had my spine knocked out of whack at a job, just by reaching too high for something. Went to a Chiropractor, he said the back had a subluxation (out of alignment) causing the muscles to contract. One adjustment was all I ever needed for this. Instant relief for me.

Q: What is this lower [right] back pain?

Today I had a sore abdomen, but now I have a pain on my lower right back.

I started taking some medication my doctor had once prescribed to me for infections, but I was wondering if someone else could else hint or guess at what it is before I freak even more.

The pain itself is felt on my lower back, to the right side, and I can sort of feel it on my lower abdomen, too; mainly on the right. My abdomen feels slightly sore, but I’m guessing it’s an infection [I'm capable of getting urinary infections or any other kind of infections because my appendix burst a couple of years back].

Any idea? Help?

A: It could be a U.T.I. (urinary tract infection). Or kidney stones. Have you tried drinking cranberry juice? Along with quite a bit of water?Look at your urine. It should be almost clear.If you’re having allot of cramps it could be stones. You should probably go to the doctor. After you’ve used the same antibiotics several times they lose the effectiveness. Sorry you are feeling yucky.

Q: What is causing my lower right side back pain?

Since early this morning I have been having pain in the lower right side of my back. Right above the hip. Then when I went to use the restroom after I went there was some blood. Then after that I went several times more and there was no blood. Anyone have any clue as to what this can be?

A: I have had pain in the lower right side of my back for about a year now, but which has got worse in the last two months. Having seen a Doctor and Physiotherapist I’ve come to the conclusion that it has been the result of bad posture, i.e. slouching in chairs, not bending knees when lifting heavy things.

One exercise you can use to improve flexibility is to lie face down on the floor and use your arms to push the top half of your body up into a sort of press up position until you feel a little strain in your lower back, hold for 3 seconds and repeat 10 times or so.

Q: I am a runner with constant lower right back pain when pressure is applied to right foot. What’s going on?

I have a constant pain for the last few days when pressure is applied to my foot. I got into my 4th mile Fri and had to stop due to the pain. I have since been using the elliptical which seems not to bother me! I am 26 yrs old and feel as if i’m to young to have back pains lol. It even pains when walking! Any advice or similiar pains from anyone else?

A: Check your feet alignment, seems one foot is causing the problem, so you need to see a Podiatrist, get your feet checked and perhaps special insoles. Good Luck

Q: Right upper back pain and a headache?

I have the worst pain in my upper right back area, close to my shoulder blade. It feels like I have to crack my back. I can’t even take a deep breath it hurts so bad. I also have a headache, I am not sure that the two are related as I had the headache yesterday before the back pain. Any ideas if the two could be related and also how to get rid of this pain, it’s terrible. I’ve had it before, in the same place, could it be something that I have to have looked at?

A: In reality, the two can very well be related, but I feel that the main problem is the one in your back and not the headache. What it sounds like is a subluxation (misaligned bone) causing nerve impingement. The subluxation could be of a vertebra in your spine or one of the ribs. The most likely muscle being affected is the trapezius muscle. This muscle attaches both in the upper back area and the back of the skull, therefore tension in this muscle can produce a tension headache as well as upper back pain. The absolute best way to take care of this is to go to a chiropractor.

Q: I Have this throbbing internal pain on my lower right back what is it?

Its not just back pain cause it feels like something internal is working over time. Its right in the back almost over my , excuse my language, butt. I have had kidney stones before but this is soo different. Along with this pain i do get, what i like to call kidney shots, it feels like a kidney stone pain but lasts about 5-10 minutes several times throughout the day everyday. Any medical professional want to give me their opinion?

A: You may want to go get checked to see if you have a kidney infection. It’s kind of like having the stones, but the pain is worse, and it lasts longer. I have had a kidney infection before, as well as stones, and I have to admit the infection was much more painful. Go to the doc. and have them test your urine for it. Hope I helped a little bit, and get feeling better!

Q: What is causing my right lower back pain?

I been having constant pain in my lower back on the right side. What can be some causes? I am afraid to take tylenol in case it is my kidneys.

It’s been bothering me for a few days now.

A: I don’t know if your pain is caused by kidney stones, but here’s a link with info on back pain and kidney stones http://www.backandneckpainresources.com/category/back-pain/kidney-stones/ . You should really go see a doctor however.

Q: Recently, I have been having upper back pain right by my right shoulder blade?

Periodically, my upper back (right shoulder blade) has been having this cold, numb sensation. I have minor scoliosis, could this be a factor? I have not been to the doctor lately, and I do not know if my back has gotten worse. What are some ways I could prevent the feeling in my back? Please answer. :-(

A: I have minor scoliosis and experience the same thing in the same place. My doctors have never suggested this as a reason although I will mention it next time I go in. I also have MS so I experience similar types of pain in other parts of my body from time to time. For me I think this is the cause. My doc said that it was from sitting for long periods of time at my job but I don’t really buy that. For you I would say that it definately seems like a nerve pain. Perhaps you have a nerve that it being “hit†or “pinched†by your scoliosis or something else. My MS causes nerve pain but I am not suggesting that you have a nervous system disorder.

Regular pain relievers will not help. Massage and streching may. Try twisting your back when you stretch and reach out to your side.

I would mention this to your doctor since this has happened more than once, although it is probably nothing to be too alarmed about.

*I hate having pain in this spot because you cannot massage it yourself.*

Hope this helps some.

Q: 37wks pregnant with a constant pain in lower right back? Woke up with a pain in my lower right back. The pain won’t go away and is there thru the fake contractions. Ice pack and tylenol don’t work. Is this just muscle pain or the start of real contractions? Heard that when you get contractions they go from the lower back to the front.

A: My contractions started in the back and like a wave to the front and middle of my belly. They came on like a wave, lasted a few seconds and then disappeared again

You should just put in a call to your doctors answering service or call center, just to be safe (ask questions)

Q: Right lower back pain that shoots down my leg?

I’ve been having a sharp pain in my right lower back recently and it shoots down my leg. It really hurts when I stand up or sit down. Now the reason I’m worried is because I’m only 15 and back pain isn’t suppose to be common in teenagers. I play water polo so i don’t know if that might be it or not.

Does anyone know what this is and how I can cure it?

A: It could be anything, nerve problem, muscle problem, kidneys or a number of other things…It is impossible to answer your question without knowing what the actual problem is. Ask your Doctor for a referral for a CT or MRI scan. The wrong type of treatment or exercises can do more damage than good and you need to have your back properly diagnosed prior to ANY treatment.

Again you do need to have a CT / MRI scan or such done before allowing ANYONE to touch your back.

Get a professional opinion from the radiologist doing the scan in his report to your doctor and then go from there.

Q: Upper right back pain, hurts to breathe and cough?

I’ve been sick with a cold for a few days but today I’m in serious pain to the right of my spine and a little above the middle of my back. It hurts to breathe and cough and pretty much do anything. Help please :(

A: you need to see a doctor the flu, influenza and pneumonia is going around!!

Q: Really bad lower right back pain at 23 weeks pregnant? My lower back is hurting on the right side it just started today and it’s now starting to hurt all the way down my leg when i walk? Any help on what it is or what to do!?

A: Your joints are beginning to get lose you just need a break and take it easy when you feel the pain lay down on your side with a pillow inside your legs and rest walking makes it more painful

Q: Neck sore?

I had a cold about 2 weeks ago and I still have the congested cough. Yesterday,I woke up with my neck sore under my left ear down my neck. I woke up this morning and it is more sore. It is starting to hurt now under my right ear. I don’t have a fever or sore throat. I looked in my throat and I have this white-yellowish patch and it is has red lines but does not hurt. Can you have strep throat without a sore throat? Has this happened to anyone else?

A: Look, if there is anything wrong with your body and you don’t know what it is, I suggest one of these two options for anytime this happens:

Q: Why are my shoulders & neck sore the morning after i drink?

It seems like everytime i drink a lot of alcohol, my neck & shoulders are really sore the morning after. Does this happen to anyone else & does anyone know why it happens?

A: Likely it is just dehydration, also a main cause of a hangover. Drink water every other drink and you shouldn’t have that problem.

Q: Sore gums on upper back left side of mouth and left side of neck sore too?

My gums near the left side of my upper wisdom tooth feel swollen and sore for the past two days and just today i noticed pain when pressing below my left jaw on my neck.

Ca this be anything serious?

A: It’s probably not serious, but requires some action. It sounds like you have a minor gum infection that is affecting the lymph nodes on the same side. The home remedy is to rinse with warm salt water several times a day and to take advil as needed. If it hasn’t markedly improve in 7 days, then go in to see the dentist

Q: Why do I get a sore neck when I sit or lie in a draught? I have been sleeping near an open window and I wake up with a stiff and sore neck. Can anyone explain the science – please.

A: I don’t think there is an association with the wind flow or temperature or anythin like that. You do want to ensure that you have the same body and neck position out in the other room as you do in your bed (where you do not wake up with a neck ache), the pillow is the same etc, good luck

Q: What is the fastest way to get rid of a sore neck?

I woke up this morning and i had a terrible pain in my neck and i knew it was from sleeping on it wrong. I would really like to know what the fastest and best way is to get rid of a sore neck is. I have a soccer game tomorrow and would like to know as soon as possible. Thank You

A: I would take some Ibuprofen or Naproxen for the pain. You need to stretch your shoulder and neck. Try rotating your shoulders backward and forward a few times. Also, tilt your head from shoulder to shoulder and forward to your chest and back a few times, do all these slowly. Applying moist heat will help a lot, too. Good luck!

Q: What is the best pillow type to support neck and to stop me getting a sore neck?

Im sick of getting a sore neck coz they are too soft and dont support my neck, they are filled with polyester fibre, any suggestions as to which pillows are best to support neck and are comfortable?

A: Use what is called butterfly type pillow which has good support for the neck. Or put a small wooden plank under the normal pillow to provide a firm support.

Q: What helps a sore neck and shoulders while sitting at a desk & when lying in bed?

I have had probs with my back & Neck for years, i’ve seen a pysiotherapist comes right for a while then goes back to being sore.

What can i do when it’s sore while sitting at my desk @ work and when i’m lying in bed I have been to a chiropractor, she made it worse so did the physio, i have been using herbal anti-flamme to help but i can’t keep on using all day

A: The problem with Physios, osteos, chrios etc is they don’t empower self healing and maintainence, they don’t tend to look to indepthly at common movement/activity patterns

Lie on a firm surface. with your knees bent and feet flat.

Raise your arms above your head and stretch to the roof.

Do little circles in each direction.

Now take your arms through full Circumductions. (big circels with your wrist just a centremetre from the floor

Then put your elbows out to the sides and let your hands ballance above them with your elbow bent at 90 degrees.

Try to find the balance point where you no longer have to hold them up, and breath deep breaths holding your lungs open.

You’ll feel all sorts of little spasm or adjustive movements in your shoulder blades, persist at this.

You can then also try lacing your fingers behind your neck and make flapping motions whilst both pushing your fingers together and your elbows out to the side.

Also knee pushups or place your arms on doorway lean forward with arms bent and push a little loading your pectorals triceps, anterior deltoids, and abdominals. This is good. You can also do the revers but this action is easier using either end of a rope hung over a tree branch or ceiling beam.

Good luck

Repeat, letme know how it goes.

Q: What’s he best way to treat stress-induced sore neck and shoulders?

I had very sore shoulders (below the shoulder blade) today. Had a massage and now my neck and my shoulders hurt a lot when I turn my head or put it forward to stand up. I’m using a heating pad now, but I wonder what other options there might be.

A: I had issues with pain after I had a massage the very first time and later discovered that part of that pain was because my body wasn’t used to the massages yet. I also learned that it’s important to drink a lot of water afterward to help rid the body of the toxins that just got built up during the massage. With my first massage I didn’t drink a lot of water afterward and that added to my pain as well as caused a urine test at one of my other doctors to show albumin (a chemical that relates to our liver) in the sample.

Something else I did for the neck and shoulder area (still do because of my fibro at times) is apply hot water to the area thru the shower.

I would also recommend getting in touch with the massage therapist to see what other suggestions they might have to help ease this pain.

Q: Is it normal to have a sore neck when you have a cold? If so, how long is it before the sore neck goes away?

A: It can happen – I don’t know why but it’s happened to me a couple of comes (when recovering from a cold). My doc gave me a sheet of neck exercises to do – several times a day. It took about 5 – 6 days to fully recover. The exercises instantly provided some relief though.

Apparently it’s the combination of a cold & some sort of physical exercise, or strain that does it (in my case lifting weights – but it doesn’t have to be that strenuous to set it off). Happens to a few people.

Q: Can pregnancy cause me to have a sore neck?

No matter how i lay, my neck and the back of my head are always sore the next day. I am only 13 weeks along, so I wouldn’t think that my spine would be causing things to be out of whack, but I don’t know what else it could be.

A: if can definitley cause you to have a sore neck and back of the head. I was about that far along when I started having pain…I called my doctor and got a Physical Therapy referral….you would not believe the difference!! even though you may not feel it, your body is getting ready for the baby and pulling on your spine…it tightens EVERYTHING up….all the way up to your neck….I got massages in PT and she loosened up all my vertebrae and then up my neck and it worked WONDERS!!! I’d talk to your doc…some PT could help for sure….haha i know it sounds funny…but I think im my PT’s biggest fan..shes helped so much! Just be careful who you go to though, make sure you go to one who specializes in pregnancy.

Q: My neck and back are sore to the touch after steroid treatment for bronchitis. What could this be?

I got bronchitis and was given methylprednisolone and now my upper back and neck are sore to the touch. Can anyone help?

A: I really think you need to call your doc’s office and ask the nurse. You could also call the pharmacy and ask a pharmacist. The reason I tell you this is b/c muscle pain & weakness and tendon & bone pain are both signs of severe side effects of medicinal steroids. Just thought you needed to know that b/c it could be something that needs medical attention. Also, you could be sore from coughing. If you coughed a lot during your stint w/ bronchitas, you will be extremely sore and uncomfortable in many areas, such as, chest, neck, throat, upper back, neck, etc. My guess is that is probably the case, but you need to run it by a nurse/pharmacist. Hope this helps, and I hope you are well soon!

Q: What’s the best way to help a terribly sore neck?

My neck has been sore for days now, and I don’t know what to do about it. I’ve had heat on it, until tonight when my pilates instructor said to not use heat and ice it instead. The ice feels good, but it still hurts like hell. What can I do to help it?

Thanks. I think a lot of the soreness has to do with stress and putting strain on it during working out, which I’m trying to work on.

A: stay with the heat i have just recovered from the chiro from hell a few mths ago ice is only used for swelling, keep the heat on it , take pain relief tablets and let it rest

Q: What’s causing my headaches & sore neck after running?

I just started a running program in an effort to lose weight and be healthy. I started running outside and I have found that those days, I get a bad headache and sore neck after running. It stays with me all day until I go to sleep. Any advice or suggestions on what causes it or how to get rid of it?

A: I’ve heard that a Budd Chiari malformation type 1 can cause this. Ask your doctor about it.

Q: During my ab workout i have a sore neck?

I always put tension in my neck during my ab workout and it gets really sore. Anyone got tips on how to stop the tension?

A: Use only your abs. You are probably doing it wrong.

Q: Can roller skating cause a sore neck?

I went roller skating and now I have a sore neck. My sister does not believe that roller skating caused my sore neck. I tend to disagree with her because that is the only thing that I did differently in my normal routine. What say ye? Yes, I did fall once on my behind. I also skated for about 2 hours. I have not skated in years.

A: sure…if you were tensing up the muscles in your neck/shoulders…maybe in an attempt to concentrate/keep your balance.

Q: What kind of exercises is okay with pinched nerve on neck?

I have a pinched nerve in my neck and sometimes i get the tingly and pain all the way to my hands. I havent gotten symptoms in months till now I went back to my chiropractor. Meanwhile I dont want to stop exercising because I enjoy it. What are some exercises I can stick with

A: Only mild stretching of the neck in all directions.

Q: Can a pinched nerve in neck heal itself?

I think I have a pinched nerve in my neck, I have pain in the back of my head that feels like a headache, and I feel it mostly when I shake my head and look up. If its a pinched nerve what can I do and if I should go to a doctor. Additional info: I am 20 years old and had this feeling in my head for 4 days. Thank You

A: Yes, almost always this is a self limited condition.

“pinched nerve†is a bit of a misnomer. The nerve may have been trapped and stretched between tensed muscles, or this could just be a pulled muscle or sprain.

In any case, the treatment is conservative. Rest, stretching, heat, over the counter analgesics if needed. You may get some benefit from massage. You don’t need to see a doctor, and you should never let a chiropractor within 6 ft of your neck.

Q: How do I deal with a pinched nerve in neck?

They tell me I have a pinched nerve in my neck…as I wait for the MRI and nerve tests do I do ice or heat to help? They did give me Celebrex…though I have no idea if it is helping! Any ideas??

A: A simple oblique x-ray can determine if you have a pinched nerve in your neck. The MRI will also show if you have a nerve impingement as well as a disc that is protruding that compresses the nerve.

If you do have a pinched nerve there are several options. Both chiropractors and physical therapists my effectively treat this and prevent surgery. Several forms of treatment may be used to eliminate the problem. Manual therapy, manual traction, mechanical traction and non-surgical spinal decompression therapy are all viable options to unpinch the nerve.

If 10 to 12 conservative treatments of chiropractic care or physical therapy do not resolve the issue, then you may seek care with a pain management physician. If you have a herniated or broad based disc protrusion, a transforaminal or epidural injection can greatly reduce the disc bulge or herniation and along with therapy treatments, can fix the problem without surgery.

6-8 weeks of conservative therapy should be exhausted before surgery is an option.

You can also get a simple over the door traction unit for around $30 at a pharmacy to use at home between your treatments. Do take the anti-inflammatory medication as well to help.

Q: Can A Pinched Nerve in Neck be Fatal?

I believe I have a pinched nerve right where I check my pulse at on my neck. When I lay down I feel like Im going to pass out. Is this fatal?

A: NO its not fatal but its agonizing and why put up with that pain, you need to go back to your doctor if you have not been already then go now. You need some kind of pain relief medication – don’t suffer like this. Its taking you almost to passing out point, so you certainly need HELP and you need it fast.

Go to your doctor, don’t delay !!

Q: Pinched a nerve in neck or back?

Last week, the tips of my fingers on my right hand were partially asleep. Now it’s changed. If I look down at the ground, the right half of my back, butt and leg instantly get the pins and needles feeling like they are falling asleep. The second I look straight, everything feels fine. Again, this only happens when I tilt my head and neck forward.

Pinched nerve?

A: see a doctor, you need an xray, a disc in your neck is on a nerve, see the doctor

Q: Can a pinched nerve in your neck cause shooting pains when you turn your head and cause tingling?

Sometimes when I turn my head I get shooting pains that go up into my head and cause a numbing and tingling feeling in my neck and head. Can a pinched nerve cause that and how can I stop this from happening, it really hurts.

A: Yes, a pinched nerve can present with those symptoms. I would see a good chiropractor because they handle this stuff every day.

Generally, most medical doctors will handle these symptoms with painkillers or cortisone injections, neither of which actually address what is causing the problem. A thorough chiropractic exam will determine what is happening and non invasive chiropractic therapies can really help, if not resolve this problem for you.

Good luck!

Q: What is the best/fastest treatment for a pinched nerve in the neck?

I had been begging my boyfriend to go camping all summer and he finally gave in, but pinched a nerve in his neck while sleeping. He has been in agony for weeks, and as a professional athlete this could really affect his work. I feel horrible! Any suggestions for quick/ thorough recovery would be greatly appreciated. Intelligent responses only please…

A: He should be seeing a sport therapists . I am sure that since he is a professional athlete he is accustomed to injury and the medics in his field that can take care him. Good luck

Q: What can be done to heal a pinched nerve in a neck?

I went to my doctor after about a week of pain in my neck. He diagnosed me with a pinched nerve, and prescribed me Vicodin, Flexeril, and Prednozone. It’s been about 4 days since I’ve been on these medications and they only seem to knock me out, making me sleep all day, not helping with the pain.

If in the end these don’t help, what more can a doctor do for me?

A: I definitely agree with the last person, but I have not had much luck with chiropractors. Perhaps you should go see a physical therapist. They can do massage to ease the pain and aid in strengthening your neck. There are different therapies that both doctors can do. Yes, look outside the box. Get off of those meds!

Q: Should I be concerned about dizziness brought on after a pinched nerve in my neck?

I’ve had a pinched nerve in my neck for about a week, and since then I’ve been slightly dizzy and light headed, and the worse it’s got, so has my dizziness. Today it’s really bad, I can barely stand, is this normal?

A: Yes, you should be concerned and I don’t think that it’s normal.

Q: Can a pinched nerve in the neck cause dizziness?

Hi, I am wondering if a pinched nerve in the neck can cause dizziness because my mom is going through alot, and now she has dizziness on top of all of it so is this possible?

A: I have had that happen to me and I can tell you, it hurts really bad and can cause dizziness real bad head aches, irritability, short term memory loss, high blood presser! You can help her out by a simple shoulder rub fallowed by a hot towel on the back of the neck. then a cold one after 20 min.s after all that she has to keep moving it and will work it’s self out! Hope all that helps you!!!! Tell her I said to feel better!

Q: Can a pinched nerve in the neck cause headache?

Apart from sleeping in terrible positions because I have two dogs that make it impossible to get comfortable at night and make me sleep all twisted, can a bad back or pinched nerve cause headaches when waking up in the morning? When I get the occasional back rub I jump because my back and neck hurt so much when being massaged.

A: Absolutely. A pinched nerve can cause anything from minor discomfort to major problems.

Ban the dogs from your bed, take over the counter pain medication to relieve the headache, and see a doctor if it lasts more than a couple of days.

Q: Can a dog get a pinched nerve in its neck like humans? My dog seems to have a pinceh nerve in its neck I couldn’t take him to the vet this morning cuz I had work do they get pinched nerves? If so do they last about 2 days or so like mine do?

A: Yes dogs can get a pinched nerve. I’d take him to see a vet to make sure that it is a pinched nerve and not something more serious.

Q: Any home treatments I can try for a very severe pinched nerve in my neck?

I am a 22 year old male and I have been suffering with very severe and chronic neck and shoulder pain for a year and 9 months now. I have gone to two doctors, had two CT scans and an MRI (they found nothing but a pinched nerve-no disk herniations or anything)…I do ice and heat therapy every night, have done every stretch imaginable, been to chiropractors, etc…I am in so much pain it is unbearable at this point-the back of my head and my neck and shoulder are so swollen it’s ridiculous and I have weakness in my right arm now. The pain gets worse with every month…it did go away for almost a month about a year ago but has been back full force for the past year…I am at my wits end with this because nothing seems to help. Any suggestions from anyone? Anything will help me at this point…plus I know it is driving my fiancee crazy that I can’t move most of the time at night

A: I know you want a home treatment, but you didn’t mention a pain management Dr. I have found them to be extremely helpful, after 9 years of pain and no dr/phycial therapy/chiropractor helped much. Once I went to pain management, I was feeling better within a few days. It was a life changing experience for me.

Best of luck to you. Hope you are in less pain soon :)

Q: What should i do about a pitched nerve in my neck& do i even have a pinched nerve?

5 days ago a sharp sting (like needles) was in my leftshoulder,and i have been taking Ibprofen and i just switched to Asprin. My whole left arm is numb from my shoulder to my fingers. i can barley stand it. Does it even sound like I even have a pinched nerve in my neck? And would I look stupid if i went to the hospital to get it looked at? Cause i have no insurance. Please give me some advice.Thank you

It’s common under high tension both mental and physical stress. Yes, muscle is squeezing a nerve from what you’re describing though I can’t diagnose I’m not a doctor. So I make no claims :) But try the above first before a doc if insurance is an issue if you believe there is no other life theatening concerns going on. Also they do a health background on you before massage so be prepared if you have medical conditions that will contraindicate (not permit) massage you’ll be referred to a doc before massage.

Q: can a pinched nerve in your neck cause tingly teeth?

i think i might have a pinched nerve in my neck, it hurts so bad my teeth are tingly and it hurts to swallow on the one side. can a pinched nerve cause that?

A: I have damaged nerves in my neck due to a cervical spine injury and I’ve never felt anything like that. Nerve pain is more of a burning pain.

Here is a link where you can ask a WebMd doctor the question.

Q: What is the sharp pain in the chest area near that heart that goes start to the upper back behind the heart?

Sharp chest pain that starts and end abruptly. Location: Near heart and leads to the back near the back of the heart.

A: Benign chest wall pain is most likely a transient and fleeting irritation of the pleura, the slippery membrane that lines and protects the lungs. The pleura are very sensitive to pain, and for reasons that are usually not clear, some momentary irritation causes a painful sensation that can be quite severe, but that quickly subsides. This condition has no medical significance whatsoever. It is very common. Most people will experience these symptoms at some point in their lives.

The pain is usually a sharp “catch†that interrupts a breath, and that returns with each breath for a few moments – then it subsides. It is not related to exercise, and generally can be localized to a specific small area (smaller than the palm of the hand) on the chest wall. It usually lasts for less than a minute, but can come back on and off for an hour or so.

This condition can be evaluated by taking a careful history. The condition is very common, and very easy to diagnose by history – as long as the doctor is aware of it and understands it. The important thing to keep in mind is that this condition is completely benign, and is not related to any medial problem or any abnormality.

Q: Is it normal to get sharp chest pain every now and then?

I get a sharp pain on the left side of my chest (seems to be near my heart) and it sometimes is so bad it takes my breath away. However, it doesn’t happen often. Usually it’s just mild.

I have chronic anxiety, but I know the difference between that chest pain, and this other kind, so I know it’s not from that.

Does it come with age? (I’m 24) Or is it something I should be worried about?

A: It could be a minor case of acid reflux (there’s a reason they call it heartburn) or gas. It’s very common. If you’re worried schedule a physical and bring it up with your doctor during the exam.

Q: why do i have a sharp chest pain when sneezing?

A couple of weeks ago I started having intense sharp pains in my chest when I sneeze. I don’t have allergies or asthma. After I sneeze, it takes a long time sometimes days for the pain to go away slowly but then I sneeze again and it hurts alot. I can feel it when I exhale deeply and extend myself like when Im stretching.

What is it? Should i see a doctor?

A: You probably have pluriesy (sp?), which is the inflammation of your lungs. Talk to your doctor, as this can be a sign of something serious. Hope I helped, and good luck!

Q: What could cause a sharp chest pain when inhaling?

What could be causing a sharp, centered chest pain when I inhale deeply? It has been occurring for almost a week now. I had a small sinus surgery three weeks ago and do not think it is an infection because I am not coughing up any mucus. I work out regularly and I’m only 16.

Any docs here?

I don’t feel like telling my parents and having to go to the hospital to get checked out.

A: the same thing happened to me when I was at holiday world. I drank like five cups of water and it felt better after that! you might be dehydrated. I get dehydrated at home even because I don’t drink enough water. try drinking water and remember to keep breathing even though it hurts.

Q: Sharp chest pain- thoughts as to what might be the cause?

I get sharp pains on my left side of the chest area near my heart. I have been having the pain for about 2 years now. I’m 27 and in fairly good health. I don’t take any mediations and I walk a lot during the day. I had x-rays done and my doctor said everything looked fine, but the pain is still there. I had my cholesterol checked this last February and it showed my HDL was 70 HDL and my LDL was 204LDL. I was told that my cholesterol was not high enough to worry about but I wonder if I’m on the fast track for a stroke or heart attack or heat disease and that’s why I have these chest pains. Can anyone give me some ideas what might be causing this? Could it be asthma? Any suggestions or advice would be greatly appreciated. Thanks!

Its not panic attacks- I’m positive. I could be stressed. I work in the IT field and I know it can be stressful. I also pick up computer equipment and I could have pulled something because I am a small woman- but wouldn’t it have healed by now? I’ve had gas before so I know how those symptoms feel and it doesn’t feel like that.

I’ve have been diagnosed with exercised induced asthma back in 94, but hadn’t had any problems in a while and haven’t had to use an inhaler in years.

Oh yea, I noticed it happens when I try to take deep breaths and it doesn’t hurt when I put pressure over the area

A: Heart burn… when this pain comes on, try some Tums and a glass of cold water. If this relieves your pain, it’s heart burn. Hope this helps.

Q: What is this sharp chest pain?

For two days now I’ve been experiencing sharp pain in my lower chest/upper stomach area. It feels sort of like heart burn, but not exactly the same. It comes and goes. Sometimes it’s on the right side, sometimes the left, sometimes in the middle. I do have acid reflux, but I’ve been taking my medicine and haven’t had any trouble with it for a few weeks. It also doesn’t feel the same as my acid reflux. I don’t know if this is something I should worry about and see my doctor or if it will just pass. Any thoughts? Thanks.

A: First of all, chest pains aren’t something to take lightly – if you’re sure it isn’t heartburn or acid reflux, you should see a doctor quickly.

A couple of years ago I had a similar experience – it started out much as you described, and over the course of three or four days it gradually got worse, until it started to feel like someone was sitting on my chest. Of course, the more I worried about it, the worse I felt.

When I finally went to the doctor, she suspected it was acid reflux, which I’ve never had before. An EKG pretty much ruled out heart trouble, but she ordered a chest x-ray just to be safe. The x-ray didn’t show any heart trouble, either – but it DID show a bit of fluid in my lungs, which she diagnosed as mild pneumonia.

After less than a week on antibiotics it went away completely and hasn’t been back.

Naturally, everybody’s different, and I’m not a doctor – but the symptoms you’re describing are very much like the ones I had.

Q: Sharp chest pain in left side under breast?

I randomly get sharp pains in my chest and they usually don’t last very long, but the last time I got it it lasted a long time and whenever I moved or exerted myself I would get a sharp pain. I’ve had this condition for as long as I can remember. What could it be? I’m 19 years old, I don’t think I’m having a heart attack.

A: you have a stomach ulcer. seek medical attention. or go look up “what are ulcersâ€

Q: I have sharp chest pain on the left side of my chest?

I have sharp chest pain that goes away very quickly. It is on the left where side my is supposed to be. I am 21 and my diet is okay, but it could be improved on. Sometimes I occasionally eat fattening foods such as chocolate and ice cream. I don’t exercise as much as I should. I mostly get these pain when I wake up in the morning. Is this a early sign of a heart attack?

A: Hardly anyone has a heart attack at 21, so it is very unlikely that you will.

It could be muscle pain. Not wearing a supportive bra, if you have medium or larger breasts, causes exactly this type of pain. Try wearing a firm sports bra for a few days and see if it gets better.

This type of pain can also be caused by weak upper body muscles and too much slumping over a computer. Try some light resistance exercises to build upper body strength and improve your posture.

Q: What is this sharp chest pain from?

Recently I have had a sharp pain in my chest, right around my heart area, several times a day. when this happens I have to control my breathing because when I breath in or out too deeply it hurts more. i have never had this before. Im a teenager and in very good health.

A: You should get to see a doctor. It may be a pulled muscle, but you should get it checked out. Chest pain is nothing to fool with.

Q: Sudden sharp chest pain about 3 inches from collarbone?

I have been getting these sharp sudden chest pain that last maybe a second, every once in a while, at unpredictable times. I probably don’t have the best eating habits, but I do drink about 4 glasses of water a day. I’m not overweight, I’m 15 and 110 pounds. Its strange, anyone help me figure out whats going on?

A: to answer the question you need more info…is it left or right sided pain. in the back of your chest or the front, 3inches below, or above your collar bone. Do you have any heart problems?(I know it might sound weird since your only 15, but sometimes heart problems can be hereditary.) When you get this pain does it seem to alter your pulse, (Does it speed up slow down or remain the same?)

honestly if it keeps up I’d just see a doc.

Q: What could cause – sharp chest pain when inhaling?

Sometimes Iget a sharp chest pain when I inhale air. It is so sharp that I can’t breathe in all the way, only half way. I have to take short breaths until the pain goes away. What’s causing this? I’m in my late 20s and I didn’t eat anything today at all (so I know it’s nothing I ate). TY. ** oh, the pain is in the front of my chest, deep inside, at the top/underneath of my left breast.

A: its an air bubble! i get it sometimes. usually it goes away. its from gas trapped inside.

Q: Brief but Sharp Chest pain on my left side?

I am a 20 year old female. I have been for the past two weeks having a sharp pain right under my left breast. It hurts really bad, but lasts for like 10 seconds. Do I need to go to the doctor? What is it? Please help. Thanks.

A: Gas build up can sometimes cause pain such as you have described (an example only).

Because of how long this has been going on for I would recommend that you see your doctor for further evaluation of the situation.

Q: sharp chest pain and shortness of breath?

So basically I’m 17 and I have been getting this random sharp pain in my left chest area. I have shortness of breath, but its like I can’t breath fully. This started when I was sick and threw up a few times, I’m not sure if it’s because I threw up, or if its something serious and should be concerned about it. The pain in the chest lasts only a few seconds, but its very sharp.

A: Relax, if it’s a sharp pain it most likely has nothing to do with your heart. Is the area where the pain is tender? If it is it surely can’t be your heart or lungs.

It sounds more like costochondritis which is an inflammation of the connection between your rib and sternum. Yes it can really hurt and make it hard to take a deep breath. You probably did it while vomiting.

There is really nothing you can do about it but a warm compress might help. It’ll go away on it own in it’s own time. God bless.

Q: Why do i get sharp chest pain near exams?

every year in the run up to gcse’s as-levels and other exams i get occasionall sharp stabbing chest pain which lasts for 10 minutes or more. I feel no pain until i breathe in past a certain point which makes my breathing quite restricted and quick movements also make me feel the pain. Something to do with stress i think, i’m a fit 17 year old male with no other health problems what is causing this and how can i fix it?

A: You are under too much stress.

Try to chill around the exam times – keep on top of the work to reduce the need to panic.

If the problem persists see your doctor.

Q: Sharp chest pain out of nowhere?

I was just laying on the couch watching a movie when all of a sudden I felt a sharp pain on the left side of my chest (where my heart would be). It’s never happened before, had no additional effects, and came out of nowhere. Any idea what it could have been? I’m only 18.

A: Your heart is in the middle of your chest, not on the left hand side of your chest. I had a similar thing happen when I was about 20, the pain was also on the left hand side, I thought the pain was related to my heart. I went to the doctor, who made an appointment for me to go to the relevant hospital. At the hospital I was hooked up to a machine for quite some time, which was monitoring my heart. In my case, the results went back to my doctor, & my heart was just fine. The pain was ‘just’ a stress related thing & I haven’t had such a pain since.

But for yourself, to find out more & to put your mind at rest, I would suggest that you go to your doctor & explain about the pain & to get it checked out if there is any question mark at all.

Q: Could right side chest pain that radiates be a warning sign that I’ve had a minor heart attack?

I was recently diagnosed with blood clots and have been having TIAs or mini-strokes. The other day I had an unexplained pain in the right side of my chest radiating down my right arm and into my back. I got very sick to my stomach. This feeling past after about 15mins, but reoccured later that night. I found rest helped.

A: With your background I would suggest seeing your doctor.

from what I hear a person can have pains just about anywhere when suffering with a heart attack. But, it could be a pulled muscle, indigestion or a host of other things.

Q: What causes brief right side chest pain?

I’m 20 years old and the pain only lasts for a second but it keeps coming back. I’m not having shortness of breath nor do I feel light headed.

A: This sounds like a muscle spasm, possibly a pinched nerve, or if it were closer to the center of the chest might even be GERD. I suspect it is likely a result of posture causing a strain on the spine.

It is hard to make a good guess, and could even be caused by a bra putting strain on the nearby nerves.

Drink more water, and avoid caffeine, alcohol, smoking and artificial sweeteners.

If the pain persists or worsens, consult your Physician.

Good Luck~

Q: What could be the cause of this chest (upper right side) pain?

Wednesday I ate and half hour later I threw up everything I ate. Thursday I start feeling a light chest pain but today it takes a whole different level: I can’t yarn, laugh, caught nor take deep breaths. I have insurance but it’s such a hassle to go to the ER and get an X-ray now (10:20 PM for me now). If you were in my position right now, would you be worried about this pain? I am in my mid 20s….

A: Definitely go into the ER something could be seriously wrong. Do you know if you have asthma (do you have chest tightness as well?)? You may also have some sort of chest infection or something. If I was in your position I would go into the ER since it is better safe then sorry, sure it will take a while but if there is something seriously wrong then that time was well wasted.

Q: right side chest pain hearts when deep breath, touch or move?

having right side chest pain since two weeks done X- rays,EKG and blood test from lugs result came out as a normal but, still pain is there even when I breath,move or touch what it could be and what could be the cure?

A: right side yes? if that is case then unlikely anyhting to do with heart as heart is on left side. this can be several things such as broken rib, lung problem, pulled or stressed pectoral (brest/chest) muscle, or even a throat problem – not just typical sore throat, more like a cancer or something. Most likely it is muscular and without muscle relaxants you may never get better, or at least not for long time. Especially if you are worried about it which makes muscle tense up. The muscle has to relax to heal. HAs to. Outside of going to doc, some streetwise ways of dealing with any strained muscle are getting drunk which relaxes you and often sleep long & relaxed too, a very good and long massage, sauna, or heat combined with relaxing music. All those have to be followed by babying the injured muscle for a while.

Q: I have right side chest pain it’s about a 5 on a scale og 1 to 10?

It’s in my back and on my chest to under my right Brest. I’ve had acid reflux lately if that has anything to do with it. My doctor did a EKG and blood test everything came out fine if you know what it help please! It also is on the left sometimes.

A: YOU MAY WANT TO KEEP TRACK OF YOUR DIET – YOU MAY BE GETTING A GALLBLADDER ATTACK – TRY WITH A LOW PROTEIN AND LOW FAT DIET FOR AWHILE IF THAT IS FEASIBLE FOR YOU – PROVIDING THAT YOU ARE NOT A DIABETIC

AND IF THAT DOESN’T DO THE TRICK HAVE YOUR MD SUGGEST A GI MD – YOU MAY HAVE TO BE DOPED THEN SCOPED (IF YOU KNOW WHAT I MEAN)

Q: i have chest pain in the centre of my chest and someitmes towards right side?

sometimes i feel pain towards right side of chest and all over, does anybody have idea what could b the cause? i did ECG & CBC test but was normal.

A: You need to get seen by a doctor! Chest pain can NOT be diagnosed on a website! Go to the doctor!

btw – whether the chest pain is on the left or right is irrelevant to heart problems, the fact that it’s in the middle is a concern and then it radiates to one side! You are risking getting uneducated answers on here, so please just go to your doctor and get checked. Goodluck!!

Q: 38 weeks pregnant and pain in right side chest when exhaling?

I’m 38 weeks pregnant and I have pain in the right side of my chest just nder my ribs when I exhale, its hard to take in a full deep breath. I should have more room in my chest now to breath since my baby has already lowered in to my pelvis as far as she can go without breaking my water. It feels as though something is under my ribs though causing pain and this is new.

A: When that sensation occurred to me; my doctor said it was due to my organs being pushed up, but it would be relieved after giving birth to the baby. Afterwards, yes I could breathe better but still had pain. Went to my chiropractor and found out that my little had actually kicked a rib out of place; he popped it back in and i’ve been smooth breathing ever since. I hope you get to feeling better soon.

Q: I have upper right side chest pain that feels like a bubble behind my chest….?

I have upper right chest pain that fels like a bubble behind my chest?….it has been going on for about two years now….. it not ever horribly severe….but somtims it hurts when i laugh or breath deep…. im only 26……??? what could this be?? i have gone to the docter , and they said my ekg was normal….and that the green stuff i cough up was from allergies….and that my lungs sounded fine

A: relax!!!!!! its nothing but just a hyper acidity, and pain due to gas pressure at chest lavel. Just simply takes some antacid before meals and walk a bit after meals.Its not dangerous if treated in time…

Q: having chest pain, right side, when inhaling?

I have been having chest pain on the right side which isn’t severe but intensifies when I inhale. I have been drinking plenty of fluids, what can this be? Im 21 and quit smoking a while ago.

A: That sounds like what is called pleurisy. Have you had a recent illness? Sometimes viral or bacterial infections can cause this in young people. Also, there is a condition known as pneumothorax which can occasionally happen as well… happens when air begins to leak outside of your lungs and into the thorax. But with a pneumothorax, you will often have trouble actually breathing and get short of breath. There are of course other causes like trauma to your ribs or another condition called costochondritis. I would just try taking some NSAIDs just in case it’s something like costochondritis. Otherwise, if you’re not having trouble actually breathing, I’d hold out for now and see if it goes away in a few days.

Q: Right side chest pain?

I have pain left side of my right breast on occasion and especially after running hard…not sure what this could be. I do have asthma but it is well controlled I would say (don’t need inhaler during run) and anxiety …but it just seems unrelated only cause usually when running All i am thinking about is running…any ideas? Its like an achy burn to the left of my right breast but not dead center of my chest…Not heartburn I don’t think Thanks for any ideas

A: I strongly suggest you see an internist for your symptoms. They can perform diagnostic testing and then properly diagnosis your current condition. At that point, an appropriate treatment plan can be determined and aid in your recovery.

Good luck.

Q: right side chest pain when eating….what could this be?

My husband tried to eat last night and got a really bad pain in his right side of his chest right where his lung is. It hurt so bad he had to go lay down with a heating pad. He said everytime he swallows something he gets pain in his right side. He hasn’t done anything to get a broke rib or anything…he’s been pretty much a couch potato for the past 3 days he’s been off work….he said it hurt so bad that if it was on his left he would of swore he was having a heart attack so I know it hurt him and he’s a big guy that can take alot of pain to bring him down ya know. Anyone know what this might be?

A: Here is a link that will help you far more than just guessing,, lol,, and that’s what all the answers you have so far are,,, nothing more than guesses by strangers,,

Q: lower right side chest pain.plz help?

i am 16 yrs old…i went 2 a doctor as i had many pimples were growing in my face.he gave me some medicine.and on the next morning i started having pain in my lower right side of chest.it pains when i swallow.the doctor said that the pimples in my face was a infection.is the pain in my chest a part of infection.plz help.it fells like a tablet being struck there.

A: You should go to your doctor.

Q: Right side chest pain 29 year old pregnant female!?

Starting yesterday morning before breakfast I noticed a pain in the right side of my chest and it radiated into my right arm. This lasted about two minutes and caused some major discomfort. The day progressed without incident and then last night I felt the right sided chest pain again for another couple of minutes without the arm pain. It went away and a couple of hours later I went to bed. Well I woke up with the chest pain but it only lasted a couple of minutes. I am concerned because I’ve never had discomfort from my chest before. I am 7 weeks pregnant and battling a mild cough and scratchy throat without fever. My regular doctor has moved and I don’t see my obgyn until Dec. 17th. Thanks for any information.

A: Honestly this sounds like gas….and after people have surgery they often have little pains in there shoulder which is gas pain…even though we feel gas can’t get up there it can…I am a nurse and I know that sounds funny but it is possible..I had a broken leg and had surgery..everything is slowed while your under anesthesia and I woke up with pain in my shoulder….Dr. said it was gas trapped. This actually has happened a couple of times ans passes. Kinda like charlie horses..why do they happen, and then for years you don’t have them?? Who knows. Also during pregnancy our cardiac sphincter relaxes that allows more acid to come up through esophagus causing some upper GI problems for women. That would not worry me..

Q: Right side chest pain hospitalized somethings ruled out. At a loss.?

I was hospitalzed for 4 days for right side chest pain just below the breast that radiates into my back. I also had some pain below my rib but not severe. A tightness in my left chest area that I did not even mention b/c I thought it was heart burn. The results show my gallbladder was fine. I have a ruptured ovarian cyst, pericaritis, and a paracardial cyst. I had them release me on friday b/c they only intended to treat w/ pain meds over the weekend and I have a baby at home. My main complaint is the pain in my right side right below the breast bone any ideas what else this could be? My son got RSV and pneumonia and I got the rsv from him. I coughed for 5 weeks. They also ruled out pneumonia in the ct scan. Could I have a cracked rib or something causing this much pain?? I am sick of hurting and do not want to take pain meds anymore. I just want it fixed!! Just want to add that I went back to the Dr. today and CT confirmed I have Pleurisy (sp?). That is the cause of my pain. The dr basically says anti-inflammatories and time will heal it. I’m on 800mb IB every 6 hours. Hope it heals up soon!!

A: Well, if they couldn’t give you a diagnosis at the hospital with access to lab tests, imaging, and the ability to examine you, then I surely can’t here, but I do want to just throw the possibility out there that you may have strained an intercostal muscle by coughing, which might cause pain like you’re describing, but the only fix for that is time…

Q: right side chest pain after eating?

i have noticed that i have been getting pain in the right side of my chest, almost like a pounding muscle pain, the pain spreads to my right shoulder sometimes, and my underarms area. This always happens after i eat as far as i’ve noticed. What could be causing this?? i am 5′10.5″ weigh 180lb’s and im only 18.

i’ve had chest pain a couple years ago, went to the doctor and the doctor said it was nothing. it was just my chest muscles developing. i dont know if this could be the same, or helpful to what im experiencing right now

I would appreciate your help

A: It could be an ulcer or a gallbladder problem, reflux, or something else altogether. Best to see a doctor for testing.

Q: what is the upper chest pain i have after a weekend of watersports?

After returning from a weekend of wakeboarding, stand up jet skiing, tubing & swimming, I have severe upper chest pain that i can also feel in my upper back. Still a week later it hurts to breathe deeply. And when i lay on my back, it hurts to get up. At the gym, it hurts to lift weights. While i run it hurts when i bounce. What muscle has been affected? Is it serious or do i just need a few weeks for it to repair itself?

A: you may have cracked a rib or severly bruised a muscle – a week later muscle soreness would have passed.

Time to see a doctor

Q: why do i get upper chest pain when i am running? and how is it prevented?

I am a 14 year old girl who does not have much experience with running and today was the first day of cross country practice. I normally get upper chest “cramps†on both sides right below my shoulder area. I get them fairly soon after i start running along with my side cramps/stitches. I have no idea why i get them. Also, does anyone have any suggestions how to get rid of them?

A: The upper chest pain may be from breathing into your chest, the respritory muscles there are small and easily fatigued. Try breathing into your stomach also called belly breathing. For the side stitches stretch your oblique muscles before running, side bends are good for this.

Q: what causes upper left chest pain in younger people? between heart and arm pit?

I am only 19 and for the past few years ive been having upper chest pain between the arm pit and heart. its sharp and pressure like. Ive had plenty of test they dont see anything they have done ekg and all of these other test. they haven’t seen anything, i changed my diet and they gave me ultram. but it really didn’t stop the pain. it just happens whenever it wants especially if i bend over more than 3 secs.

A: Well, upper left chest pain with sharp pressure can be a sign of a few things, most of them cardiac problems. I would imagine that they’ve done an EKG and other tests, so I would imagine they would have seen something. Are there any other specific symptoms, any patterns on when you get your chest pain and what helps it.

Q: What does constricting chest pain to the upper right of the chest mean?

I’m asking this question for my Dad whom is having constricting upper right chest pain and doesn’t know how to read it as he has never experienced it before. What does that sound like to you?

Thanks!

A: Could be anything from a pulled muscle, heartburn (acid reflux) to even a pinched nerve in the neck/back (cervical spondylosis). But for a proper diagnosis, he should see a doctor. Good luck!

Q: I’m 22 and have been having upper back and chest pain for the last 3 days. Sometimes dull, others stabbing. ?

For the last 3 days I have been experiencing chest pain and upper back pain. At times it also has been radiating under my arms. Part of the time it is very dull, and others it is stabbing. And then there are times where I have not pain at all.

I’m 22. I have had a cold recently. A little cough, but that has gotten better over the last few days. Any suggestions or answers would be greatly appreciated.

A: I am not a doctor, but my husband was having pain in his chest, and back. He thought he was having a heart attack. He had recently had a cold also. We went to the e.r., and after a chest x-ray, they told us he had pleurisy. It is mucus, or fluid around your lung causing pain. The doctor explained it to us like this. He said it was like pneumonia, only instead of being inside the lung, it’s outside the lung. I suggest you get to a doctor, and find out what is wrong. Normally it isn’t your heart if the pain comes and goes, but you never know.

Q: Upper chest pain near heart and shooting pain in back?

A: When I would tell this to my mom she would always tell me that I had a bad case of gas! Try taking some Gas X to see if the pain subsides. A big sign that it is gas is that you keep burping. If the pain does not go away see your doc.

Q: I have upper chest pain near my right shoulder ?

Im not sure if its my shoulder or something else. It hurts when i move my arm sorta but sometimes if i breath. I thought it was my heart at first but my hearts on the left side. I smoke but im quieting. Anyone have any ideas on what it might be? I know my shoulder hurts sometimes from the way i lay on the couch or my bed, and my dog yanks my arm when i take him outside on the leash.

A: It could be a lot of things, nerve damage, muscle spasms, respiratory infection, etc, it’s hard to tell without more information. How old are you, and are you physically active? Is the pain sharp or dull, and does it seem like a shooting pain? Are you on any prescribed drugs, and are there any knwon medical conditions in your family?

Q: 6 weeks ago I got caught in an armbar. I have shoulder, neck, and upper chest pain. What could it be?

I got an X-ray and there is a slight gap between my clavicle and my arm where they connect. Has this happened to anyone else and what did you do for it?

A: go to the hospital

Q: What should I do about my chest pain,.upper abdominal pain(left and right sides)and general feelling sick? I’ve had my gallbladder removed 8 mo’s ago,but that didnâ€t alleviate my pain. On a scale of 1-10, I say its a 10!!!! The chest pain is getting worse and it feels like I am having mini-heart attacks. On several trips to the ER they have found nothing wrong and I still feel like I am literally dying, I have no energy and I feel completely ran down. During the “attacks†I feel hot and sweaty, my heart is pounding,my head is pounding,pain shoots up through my chest into my shoulders, and neck and my right arm feels really heavey and strange feeling, plus I feel nauseous. I have blacked out before because the pain was just so intense and it felt like I couldn’t breathe. I have had chest x-rays,EKG’s,and a chest CT scan and nothing has been found. I don’t know what else to do. My family life, career,and social is falling apart because I can’t do anything because I feel so horrible!!!!! Someone Please Help!!!!!!!!!!!!!!!!!!!!!!!!!!!!!!!!!!!!!!!!!!!!!!!!!!!!!!!!!!!!!

A: I’M SO SORRY THAT YOU ARE FEELING SO POORLY. TO BEGIN WITH I WOULD GO TO SEE A CARDIOLOGIST AND UPDATE THEM ON ALL THE THINGS THAT YOU HAVE MENTIONED. I AM SURE THAT THEY WILL NEED TO DO A FULL WORK -UP ON YOU AND THEN IF THEY FIND NOTHING CAN OFFER YOU SOME ADVISE ON WHAT ELSE NEEDS TO BE DONE

Q: What should I do if I have unexplainable upper back and chest pain?

For a week I’ve been having this horrible pain in my chest everytime I swallow. For 3 days the pain went to my back. So now I have both upper back and chest pain, everytime I swallow. I went to my doctor 3 times and to the Emergency Room once, but they have no solution. I tried Motrin, Mylanta, Advil, and Ibuprofen. Yet, no medicine works. Is there anything I can try or do to relieve this pain? I have not been able to eat or even drink a simple thing like water without pain.

A: Have you been vomiting?

Did you recently choke on something?
Stop the Advil/Ibuprofen (same thing) and take Mylanta at least 4 times a day (after meals and before bed). Eat 6 small meals a day instead of 3 large ones and try to always have something in your stomach. Thick foods may be easier to swallow (milk shakes, puddings etc.).Stay away from acidic, spicy and greasy foods.

Get a referral for a GI (gastrointestinal) specialist.

Q: What could be the cause of this chest (upper right side) pain?

Wednesday I ate and half hour later I threw up everything I ate. Thursday I start feeling a light chest pain but today it takes a whole different level: I can’t yarn, laugh, caught nor take deep breaths. I have insurance but it’s such a hassle to go to the ER and get an X-ray now (10:20 PM for me now). If you were in my position right now, would you be worried about this pain? I am in my mid 20s….

A: Definitely go into the ER something could be seriously wrong. Do you know if you have asthma (do you have chest tightness as well?)? You may also have some sort of chest infection or something. If I was in your position I would go into the ER since it is better safe then sorry, sure it will take a while but if there is something seriously wrong then that time was well wasted.

Q: What is causing the upper back and left chest pain I have been having?

I am only 14 years old, but recently I have been having shooting and throbbing pain in my upper back, in my chest, and on the upper left part of my body. I know that it isn’t a heart attack or anything like that but I need some reassurance, and whether I should see a doctor immediately. no I haven’t been doing anything really out of the usual as far as exercise goes.

Also it just happens at different times of the day. It seems as though it comes and goes on its own

A: Have you been doing strenuous exercising or work that could have caused the pain .Does it come at certain times of day or activity ?

I think if its been going on for awhile you should get some advice from a doctor to ease your mind.

Q: Can upper back pain and chest pain/muscle pain come from coughing too much and for too long?

I started with a cold/cough about a month ago. About 2 weeks ago, the cough was still going and my chest wall and muscles were hurting pretty bad. I went to a doc who told me it was just from all of the coughing. Since then, the cough is still here, and now in addition my upper back is hurting a lot when I cough. Should I go back to the doc?

A: If the pain becomes unbearable I would go for a doctor visit. BUT you can experience all your symptoms from coughing.

You can break ribs from coughing hard. When you start to cough really hard, hold a bed PILLOW in front of your chest. It will lessen the pain from the cough.

Drink plenty of fluids. Believe it or not, the fluids help flush your body out. (water or juice only)

An Xray would clear your mind. If you continue or start running a temperature over normal I would phone the doctor.

Q: Pain in left upper chest when running for a while? After running for a while I tend to get a sharp pain in my upper chest. I don’t stop and keep running because it happens often and I think it is normal. What could it be?

Not out of shape and run on a consistent basis. Has been there for years.

A: i’ve had the same thing happen to me a lot. im not sure what it is but ive always ran through it and been fine

Q: Is lower abdominal pain and excessive hunger a first sign of pregnancy?

For the past week I have been having lower abdominal pain on and off. It isn’t unbearable but it is a little uncomfortable. I ran for a bus yesterday and the pain got a bit worse and hurt to sit down. I also have noticed I am hungry a lot more and eat a lot more too.

Could this be a first sign of pregnancy?

A: Yes, definitely could be! I know at the beginning of my pregnancy I was hungery all the time. It only lasted about two weeks though & any weight I put on then has come of already & i’m only 16 weeks gone now.

Q: What causes lower abdominal pain in women?

I’ve been having lower abdominal pain, spotting, and nausea for about 2 weeks now. I’ve been on the birth control MIrena for over a year now. I have no clue to what’s going on.

A: it could be lots of things but you should never play around when it comes to your reproductive system. esp if you want to have kids in the future. you could have a cyst or a condition called endometriosis.

Q: What can cause sporadic and excruciating upper and lower abdominal pain that causes frequent urinating in men?

My partner has been suffering from this sporadic and severely painful bouts of abdominal pain since he was young but as they were only every now and then and were unpredictable he just put up with them. But of late they have been getting worse and have been keeping him awake all night as well as rendering him basically immobile. They can be in the upper or lower abdomen and cause him to urinate more frequently. What can cause this type of pain? I have no idea what to do for him!

A: It is impossible to guess and too important to give the wrong information.

It could be anything from a serious kidney problem to constipation.

The frequency of micturition could indicate a bladder or kidney infection or could be caused by stress. Please tell him he must make an appointment to see his doctor.

That is the best thing you can do other than give comfort and support during these episodes.

Notice if anything in particular precedes them, a certain food, stress for example

Q: 6weeks pregnant and having persistent lower abdominal pain, should I be worried?

I am not bleeding or spotting but have had this persistent lower abdominal pain for 3days now. Its not on either sides, but feels like PMP. Sometimes it goes away when I am walking or doing other things, but when I lie down it comed back. Has anyone had this kind of experience, is it normal? Should I be worried? I am going for scan tomorrow, otherwise I’ll go crazy with worry. Help!

A: I am 6 wks 4 days and I’ve had cramps for about a week and a half at least. Actually went into the ER doctor about a week ago because they were so bad (but I think I freaked out a bit too- as this is my first pregnany). Went in for a scan a few days later and doctor said everything was fine. If you’re not bleeding I wouldn’t worry too much- Take some Tylenol (no Ibuprofen). Also- I was even spotting, mostly brownish/pinkish and the doctor said that was even normal. They also told me that if you are having pain on one side (although I know you said you aren’t) that it could be from the ovary you conceived off of. Just relax, the stress made mine worse I think and after the doctor did the scan and told me I was ok- I’ve felt quite a bit better. Also, drink lots of water and stay hydrated and rest. Congrats and good luck! Since we’re about on the same week, email me if you have any other questions! Have fun- hope you feel better

Q: I am 39.5 weeks along and this morning am experiencing lower abdominal pain. Could this be the start of labor?

The pain at times feels like sharp twinges, but it is not constant. My uterus does NOT feel like it is contracting. But are these pains/cramping an early sign of labor? (The pain is in my lower abdominal area but is also reaching down to my vagina) Thanks.

A: Sounds like you’re getting ready to start labor…YAY!!! I had the same thing. Except I didn’t realize I had start labor until I was driving home from the store and the pain was longer than normal and I decided to time the pains. My uterus didn’t really feel like it was contracting too much, it was more of the pain that radiated from the back to the front for me. I didn’t realize it was tightening until I put my hand on my stomach and I FELT it get tight. But you probably are going to the doc every week now, so I would as them.

Q: What could be causing my 5 year old little girl’s lower abdominal pain?

She has been complaining of lower abdominal pain on and off all day today and it is starting to worry me. It’s tough with kids because its hard to tell if they are really in pain or not so much. Does anyone have any suggestions for this?

A: This could be gas or possibly a UTI. Take her to her Dr.

Q: My cousin is 11 weeks, 2 days pregnant. Is it abnormal to experience lower abdominal sharp pain on one side? 11 weeks, 2 days pregnant. Experiencing bad sharp lower abdominal pain on one side. No cramping, nor bleeding. Just that sharp irritating pain.

A: If she has had an ultrasound and they know the pregnancy is in the uterus than it is likely just muscles & ligaments stretching. If she has not had an ultrasound she should probably see the doctor to be sure. At any rate it never hurts to call and ask that is what they are there for:)

Q: Lower abdominal pain in the beginning of your second month of pregnacy, What can cause it?

Ok, so I am having lower abdominal pain and it hurts really bad when I move. What could be the cause? I dont think it is cramping. Any ideas? Please help. Thanks in advance!

A: I had this with a couple of pregnancies. My doctor said there is a ligament or muscle that runs from hip to hip across the lower belly and sometimes just the pressure of a baby or uterus on it can cause pain. If it continues, I would advice a maternity belt which helps support your belly as you get further along. Also, if this is your first, sometimes the first one is painful because these muscles haven’t been used in this way before. Hope all goes well. Good luck.

Q: what is cauding my severe lower abdominal pain?

this has happened several times now, where after a sudden urge to use the loo has triggered a sudden lower abdominal pain which is so severe i have been rolling around on the floor, sweating, thirsty, and hyperventilating. this lasts for about 3/4 hr and then eases, but i am left with a very tender abdomon which lasts for hours and feels that it could start again at any moment. the pain is central and feels like a gyny type pain. Any ideas?

A: Hey, I’m NOT a Dr. but how about checking out a Urologist or a Gastroenterologist? Both are good Doctors in their own fields of expertise so look up in the phonebook for each one of these Doctors, tell them your problems and see what they say or there is always the Emergency Room in the closest hospital.

Good Luck.

Q: After I masturbate I get gas a headache lower abdominal pain I also feel fatigued for a day or 2 any ideas?

I’m 16 years old male and for quite awhile now I’ve noticed that after I masturbate I get gas lower abdominal pain a headache and feel fatigued for a day or two afterwords. Also I don’t know if this is related but I wake up frequently to use the bathroom at night. Any ideas why this might be happening or what I could do would be helpful.

A: it could be a number of reason from bladder infection too stress or nerves .so it would be best for you too visit your doctor and let him send you for some test. the sooner you make an appointment the sooner you will know the cause and get proper medication for it .it really sounds like your bladder if it bothers you that much as it is the weekend go too the emergency room

Q: During sex recently I have begun to feel intense lower abdominal pain that sometimes lasts for days.?

My boyfriend and I have had a monogamous sexual relationship for over 4 years and recently like the past 6 months I have been experiencing severe lower abdominal pain during sex that sometimes lasts for days at a time. I found out that I had an ovarian cyst on my right ovary. COuld that be the source of the pain?

A: That sounds like it could be your answer, or he could be hitting your cervix located inside of the vagina. (This part dialates when you give birth…And contractions hurt!) If you are having sex missionary, try a few different positions so that your cervix is out of harms way. Try doggy style, or laying down on your stomach with your boyfriend on top.

Q: Lower abdominal pain caused by lifting toddler?

I have been experiencing a dull pain in my lower abdomen for a day or two at a time for several months now. The pain is the worst in the morning. I have not changed my diet, nor has any of my female cycles changed or been irregular- the only thing I can think of is my nephew who I babysit. Hes quite heavy (40 lbs about – and I’m 20 years old and 120 lbs) and I sometimes lift him, for example yesterday he threw a tantrum and I lifted him up and he proceeded to struggle with me and kick me, today I woke up with the pain again- my question is this- can this kind of lower abdominal pain be cause by lifting a heavy child and/or the kicking, even though i don’t have any back pain?

A: You could just be reaggravating a strained muscle, or I’m sorry to say, you may have a slight herniation. Go to the doctor and mention the possibility and see if they’ll take a look. It may be nothing, but if it is a herniation, the sooner you know, the less damage you’ll do yourself, and they may have to fix it!

Q: Persistent abdominal pain (lower right part), but not appendicitis?

I’ve had an annoying pain and feeling of burden in the lower right abdominal part for a few days. When I run my hand over my stomach I can feel something like a thin hard bump that is nowhere to be found in the left part of my stomach. Sometimes in the evening I have a slight fever of 99.5. I went to the doctor, they ran several tests and all, including the white blood cells account came out normal (the doctor never bothered to palpate my stomach though), so I was told that nothing particularly is wrong.

Is it possible that despite the normal blood cells I have appendicitis? What can that bump possible be?

A: I don’t know what the bump might be.

The pain could be your appendix or ovary perhaps. If your white cell count was okay then your appendix probably isn’t infected.

Are your bowels okay? Are your periods okay? Have you twisted or injured yourself somehow?

Unless you’re very thin I doubt that you’re appendix would be palpable just by running your hand over your abdomen – the appendix lies deeper down in the structures.

If it doesn’t resolve then you’ll need to revisit your doctor and ask for an abdominal ultrasound or at least a physical examination.

Q: Lower abdominal pain and some other things?

I have been having lower abdominal pain , tender breasts , I have been using the bathroom like every 20 minutes , I haven’t started my period and I have also lost 10 Ibs in two days. What I was wondering is if these are signs of pregnancy? Cause I really need to know.

A: I def. had all of those symptoms when I became pregnant, all three times, however each woman is different.

If your period is late, you should be able to test accurately positive/negative for pregnancy.

If you’re going through anything stressful or if you’ve been extra active, that can also delay your period and kinda make your body ache.

I would try taking a preg. test, just in case.

Q: I am worried about the severe lower abdominal pain i felt since yesterday.?

My lower abdomen is painful, I can not even walk straight, talk loud or anything that would connect to my abdomen. I have fever of 39.1 degrees celsius, it’s on and off. I took paracetamol and other pain reliever, but it seems there is no effect to my abdominal pain. I am already finished with my monthly period just a week ago.

A: see a doctor

Q: What is the cause for severe muscle & joint pains, abdominal pain, blurred vision & thrush at a time ? What is the cause for severe muscle & joint pains, abdominal pain, blurred vision & thrush at a time ?

please reply me…

i am facing the above symptoms from almost 3 weeks.. does these are the indications for any major disease..

A: A combination of blurred vision and thrush are classic symptoms of diabetes, along with thirst and excessive peeing. I cannot say regarding the other pains you mention but I don’t believe they are generally symptomatic of diabetes. Your very best next move is to delay no longer but go see your doctor asap and have yourself checked out. If you do have symptoms of any diseases/illnesses, then the sooner you get a diagnosis the sooner you can begin treatment.

Q: How can you tell the difference between joint pain and muscle pain?

Is it joint pain or muscle pain that stops almost immediately when you quit moving or change positions to rest or get relief for the suddenly very sore area?

A: Very seldom, if ever, are you going to feel joint pain. There are no pain receptors in the joints or ways of sending messages. You are really feeling the pain in the muscles around that joint, ones that have become inflamed or are having pressure put on them. So the real answer is it’s all muscle pain.

Q: What tests does a rheumatologist do on anyone with raynauds & muscle & joint pain?

got a 1st appointment with a rheumatologist in a few weeks following a referral from my GP, I have raynauds in both hands and a few other symptoms, muscle & joint aches all over daily, etc…RF & ANA blood tests were normal, what other tests can I expect from the rheumatologist,

A: Wondere, of course they always draw blood work and he’ll be looking at your white blood cell which is always elevated when inflammationn is involved, which it would be in the case of RA,he/she will probbally check out your sed rate and ana antibodies maybe indicating some form of lupus or auto-immune diseases, you could have fibromylgia , you could have any of the above plus more, go to the medical westirt ( web md ) and see if your symptoms fit with lupus or just look at auto-immune and it will have loads of info on just about any disease. Hope this helps. Rita V

Q: How can ease & soothe muscle/joint pain?

since yesterday i have had muscle/joint pain in my right wrist it started as nothing became a dull ache got a little sorer so i put some deep heat on it but that made the pain worse and some things very uncomfortable to do i haven’t knowingly done anything that would cause this is there a good way to ease the pain

A: Hot bath, Raljex, Gentle Stretching.

Q: Is severe muscle/joint pain permanent if caused by Fosamax? Hip/muscle & joint pain began after 2 months on 70 mg(month) of Fosamax & has gotten progressively worse after a total of 6 months on the drug — stopped prescription this week.

A: time to talk to the doctor who prescribed it to you about this.

http://www.yourlawyer.com/topics/overview/fosamax

seems these things you are suffering from are side effects from this medication.

Q: is it muscle or joint pain in my right wrist?

today morning when i woke up all of a sudden my right wrist and my hand started paining whenever i grabbed anything. its around the joint…but if i touch or hit my hand hard i feel nothing but if i close hand hardly or grab a pencil to write something my wrist pains. its like some muscle pulling pain….and i cant even write properly at this moment please help me i want to what should i do for this…thanks.

A: Could be carpal tunnel syndrome.

Q: How to tell the difference between joint pain in knees and muscle pain?

I do heavy squats about 3 days a week and sometimes I can’t tell if the joint is in pain or the muscles surrounding them are. Does anyone know anything I can do to help me figure it out?

A: muscle pain from squats is usually in the thighs and butt there arent really knee muscles, sounds like ur joints

Q: Bad headache, stomach pain, extreme fatigue, muscle/ joint pain and dizziness/nausea?

Since March of this year I have been have been experiencing pain in the left side of my stomach, headaches, fatigue, muscle and joint pain and dizziness and nausea. I have been to doctor after doctor and they have been no help as of yet. I’ve even been to Johns Hopkins with no luck. If you have any possible idea what it could be I would REALLY appreciate it! Thanks!(:

A: We have Top quality Health and fitness products that will meet your specific need for total balanced nutrition because ours are potency guaranteed! By pharmaceutical standards Please contact us and we will send you this information

Q: What is the compond of Glucosamine? How does it help joint and muscle pain relief? Is it vitamin or what?

At my pharmacy there is product herbal based; Natural Support Joint & Muscle Comfort Support. Pharmacist said no known success of this. Suggested Gluscosamine for repair cartlidge; many brands available. Which is best?

A: Glucosamine is an amino acid that is usually taken from shellfish (the slimey lining) its C6H13NO5 It may help to stop or reverse joint degeneration by stimulating the production of hyaluronic acid and proteoglycans which are the essential building blocks for forming and repairing cartilage. Research has been pretty good and unless you are alergic to shellfish or diabetic (its a type to sugar) its usually fine to take.

Boots, Tesco etc should all be about the same quality, they have a reputation to keep up, although as its not a drug its not regulated and products do vary. You can get it on prescription by your GP which should ensure quality (you hope!)

I’m firmly in the ‘give it a go’ camp, 1500 mg a day and give it at least 3 months to help your arthritis

Q: Have any one use the spray for joint & muscle pain; does it work?

I just want to know if it work’s before i waste my money; the product simply says to spray in the mouth and watch your pain dissolve; sounds to good to believe.

A: I am using Bio Freeze it is a roll on, and there is a jell as well. It workks fine, and lasts for a long time.

Q: How do you know when to apply heat or ice down an injury,muscle / joint pain etc.?

A: If its muscle pain heat should be applied and if it is joint pain you dont really need anythin except maybe pain killers. Ice should be applied to stop an injury swelling and less inflamed.

Q: What causes joint and muscle pain in the legs?

I’ve had severe pain in the ankles, knees and hips – and muscle pain in the upper leg since May.



Vitamin D levels are okay.
Lupus tests negative
Thyroid levels normal
but parvo in February

Prescription NSAIDS bring no relief – need a referral to a specialist, but any ideas what this might be other than fibromyalgia?

I like to do a lot of research BEFORE seeing a doctor – you get all of 5 minutes of their time, so I’d like to know what’s going on so I can ask any important questions or give any helpful information.

A: Your leg pain may be claudication. Claudication is most often a symptom of peripheral arterial disease, a potentially serious but treatable circulation problem. Walking may actually become a key component of your treatment program — and a way to return to an active lifestyle.

In peripheral arterial disease, the arteries that supply blood to your limbs are damaged. This damage is often the result of atherosclerosis. Atherosclerosis can develop in any of your arteries, especially those in your heart. When atherosclerosis affects your arms and legs, it’s called peripheral arterial disease.

Atherosclerosis makes arteries hard and narrow. That’s because the arteries get clogged with clumps of fat, cholesterol and other material, called atherosclerotic plaques. These plaques can make arteries so narrow that less blood can flow through them.

One of the classic symptoms of peripheral arterial disease is pain in the muscles of the legs, especially the calves. This pain is called claudication, also known as intermittent claudication. Claudication can also occur in the arteries of your arms. Claudication is sometimes considered a disease, but technically, it’s a symptom of a disease.

It’s very possible that its Fibromyalgia as well. Here is a brief list of common symptoms of this disease:

Chronic pain throughout the body
Burning, numbness and tingling
Tenderness when pressure is placed on or around the neck, elbows, hips, thighs and knees.
Sleep disorders
Chronic fatigue or exhaustion
Depression
Anxiety
Facial Pain
Jaw Pain (TMJ)
Memory Loss
Irritable Bowel
Tension or Migraine Headaches
High sensitivity to foods and medications (allergic type reaction).
Minimal tolerance to heat and cold
High sensitivity to bright lights and sounds
Hair Loss

Because symptoms develop gradually, this disease is often misdiagnosed. It is often diagnosed as a repetitive strain injury; sleep disorder condition, irritable bowel syndrome, rheumatoid arthritis or any other type of medical problem. Based on the American College of Rheumatology a person is diagnosed with Fibromylagia when he or she suffers pain throughout the body for at least three months and has 11 out of 18 tender points present.

I hope this helps.

Q: What could cause joint pain and also muscle pain?

This summer my mom started feeling pain in her right elbow. Later her whole arm would hurt so the doctor said it was probably Tendinitis so she gave her a few exercises to do. Now her joints in her wrist, knees, ankles and toes sometimes hurt. Doctors still don’t know what it could be. Any ideas??

A: Sounds like some type of Arthritis, like Rheumatoid Arthritis or something like that. I would tell you rmom to go back ot her doc and mention how it has spread to other joints and all that. They may want to do some blood work to make sure nothing else is wrong.

Q: What would cause sudden cramps, diarrhea, joint pain, muscle aches and sinus pressure?

Has been happening on and off for the past 3wks to my husband. He would be fine one minute and the next would have all of these symptoms. What could this be? He has a history of testicular cancer and lymphoma.

It has been going to my husband for 5wks and he has lost 23lbs. His mother was just diagnosed with Crohns disease. Could it be that or related to that?

A: wow, he has all that on and off for 3 weeks.

With his history, he obviously has a weakened immune system, so what has his dr say is causing this ? or did you really think that asking on here was better than taking him to his dr ?

Q: Hey is it true if you have problems with one of your organs you will have joint or muscle pain?

For example if this person has liver problems and it makes their spleen a little inflammation and they are having very bad pain on their left subscapular, you know on the left shoulder blade. Can this person have muscle pain because of liver problems, is it really possible? And this person never injured their subscapular. Please help.

A: It is possible for a pain in an organ to be felt elsewhere in the body. This is called “referred pain.â€

Liver problems do not often cause pain, nor do they cause spleen irritation, normally. Many diseases cause both the liver and the spleen to increase in size.

Q: Seeking info re: administered spanking for chronic pain relief?

I suffer and contend with chronic pain in arm, shoulder & elbow for many years.

Doctor advises the only solution to my pain management is surgery.

I read or heard somewhere that administered spankings may provide relief from chronic pain.

I have tried heat, cold, massage & Chiropracter to address the chronic pain and these methods have helped. However overtime the effectiveness of these methods have diminished. At this point I will consider any method to relive the pain and to avoid surgery.

A: Well, I’ve known spanking to achieve excellent therapeutic results – it’s a great stress-reliever, for a start – but I can’t quite see how it would relieve chronic arm pain.

On the other hand, you might well try giving your g/f (or b/f) spankings as a therapeutic form of arm exercise. And even if it didn’t work, it could be a lot of fun….

Q: How well does Hospice patients receive pain relief? If they are in chronic pain all the time?

I am considering going into hospice or I think it’s called Pallative Care, but, I just wanted to know how they help with pain, when other pain meds, such as Ibuprofen, Aleive, the gammet of OTC meds and meds such as Vicodin won’t help anymore?

A: you’ll end up on the something like IV morphine. Hospice is very good at managing pain, they work with the doctors to help the patient.

Q: On a slightly chubby body, where is the best place to put a “duregic†patch for chronic pain relief?

I have problems with the patches staying “stuck†on. They want to keep falling off. Help!

A: You have been given some great answers but I wanted to comment on your difficulty with keeping the patches on.

If you are receiving the name brand patches the ones that say DURAGESIC. you can call Janssen and they will provide at no cost to you Bio-clusive covers which can be applied over the patch to keep them is place. The number to call is 1-800-janssen or 1-800-526-7736

You can also visit your local drug store and ask for Tegaderm. this is similar to the Bioclusive covers. These were designed to protect an IV site but they can be used to keep the patches in place as well. They are a bit expensive but do work very well.

Q: What is the best non-narcotic or narcotic pain relief for chronic hip and leg pain?

The pain stems from an herniated disc at L4 and L5. The pain feels like the flu but for about 4 months. I have an appointment today and was wandering what I need to ask for.

A: oxycontin in my opinion would be the best pain relief option for you. It is a sustained release form of the narcotic oxycodone, and I think it sounds like you need around the clock pain relief.

Q: Has anyone gotten relief from chronic pain by taking HYDROXYZINE PAM?

I have been diagnosed with many different things the last on being PNE pudendal nerve entrapment. Have had chronic pain since 2001. Any comment would help me.

A: Many anti anxiety drugs are prescribed for pain. Especially Chronic nerve pain. I was prescribed a differant type or brand (how ever that works??) and had alot of success.

Q: A Microchip for Chronic Pain Relief ?

Does anyone out there suffer from some type of chronic pain? Check out this microchip technology… http://rbaloy.cieaura.com/home/pure_relief.html

A: have a good read of the “disclaimer†in really tiny hard-to-read print right down the bottom of the page – just so you’re more likely not to see it. If these things are only sold for “learning, self-improvement and simple relaxation†and if nothing in their sales pitch should be “construed as a claim or representation that these products are intended for use in the diagnosis, cure, mitigation, treatment or prevention of disease or any other medical condition†– then that web site is designed to milk desperate people suffering pain of money on a regular basis, and no better than those who rip off cancer sufferers with promises of miracle cures.

Q: Anyone have ideas for chronic neck pain relief?

I have had this chronic neck pain for a few years now. I have gone to neurosurgeon, chiropractor, ct scans, mri’s, amytriptoline, feldine, and still continute to suffer from it. It’s all day everyday,but is present when I “moveâ€. It’s located on the right back-side of my neck just at the top of the spine. They have found nothing in the x-rays or the scans. Chiropractics and the meds did help a little, but I didn’t cure it. I am looking for all and any help possible. Thanx so much to anyone who answers!!!

A: I broke my neck in 98 & never showed up on x-rays for 18 months. I can relate to what Ur going through. Till you get to a health supply store to get a herbal tube U shaped sack that U microwave & place on your neck try this. Get a towel, get it damp & nuke it then place around your neck. Simple exercises like slowly moving head L – R & same thing up & down. Only move till before it hurts. If your Doc is not giving U pain meds, try a methadone clinic. What U can get on line will not be strong enough unless U get soma. Pain meds are not that strong. Tramadal is about the strongest you can get. Be careful about ordering from overseas. Ask Ur doc for trigger point shots, these work wonders for about 3-4 weeks. There is the kind you get while in front of an x-ray machine that go in your disc, these last for months, but they do hurt. As long as you have been in pain you should get into pain Mgt. That’s where you get relief, get Ur doc to refer U to one. I hope I’ve helped you, I know what it’s like.

Q: Chronic Back Pain Relief – Which Treatment For Chronic Back Pain Works?

Looking for some effective, natural and non-surgical cure for chronic back pain.

A: These are the Top 7 non-surgical, natural options for chronic back pain relief: The following sample questions are similar to those on the examination but do not represent the full range of content or levels of difficulty. The answers to the sample questions are provided after the last question. Please note: Taking these or any sample question(s) is not a requirement to sit for an actual certification examination. Completion of these or any other sample question(s) does not imply eligibility for certification or successful performance on any certification examination. To respond to the sample questions, first enter your first and last names in the boxes below (this information will not be recorded; it is strictly for purposes of identifying your results). Then click the button corresponding to the best answer for each question. When you are finished, click the "Evaluate" button at the bottom of the page. A new browser window will open, displaying your results, which you may print, if you wish.

1. Which statement indicates the development of opioid tolerance?

Larger doses of opioids are needed to control pain compared to several weeks earlier.
Stimulants are needed to counteract the sedating effects of opioids.
The patient becomes anxious about knowing the exact time of the next dose of opioid.
The patient no longer experiences constipation from the usual dose of opioid.

2. When assessing a patient's cultural beliefs about pain, the pain management nurse asks about:

a family history of pain.
home remedies used to treat pain.
the frequency of visits to health care facilities.
the patient's dietary preferences.

3. When assessing a patient for possible side effects related to acetaminophen (Tylenol), the pain management nurse asks the patient about the use of:

alcohol.
marijuana.
opioids.
tobacco.

4. The pain management nurse observes a male patient with complex regional pain syndrome not wearing his right jacket sleeve. The patient reports intense, right arm pain on light touch. The nurse recognizes this pain as:

allodynia.
hypoalgesia.
neuritis.
paresthesia.

5. A 45-year-old male patient reports pain in his foot that moves up along his calf. The patient states, "My right foot feels like it is on fire." The patient further describes that he has no previous history of injuries or falls, and that his pain started yesterday. Which components of pain assessment has the patient reported?

Aggravating and alleviating factors
Exacerbation, and associated signs and symptoms
Intensity, temporal characteristics, and functional impact
Location, quality, and onset

6. A 53-year-old patient, who is receiving ibuprofen (Motrin) 400 mg, twice a day, for chronic, low back pain, develops lower extremity edema. The pain management nurse suspects that the edema is caused by:

a decrease in renal function.
a low creatinine level.
an increase in glomerular filtration rate.
an increase in plasma proteins.

7. A distinguishing feature of a cluster headache is that it occurs:

bilaterally.
globally.
occipitally.
unilaterally.

8. A 73-year-old male patient with cancer is in the hospital for pain control and rates his pain as a 12 on a numeric pain rating scale of 0 to10. Thirty minutes after administering IV pain medication, the patient reports no pain relief. The pain management nurse calls the physician for additional orders for pain medication. The nurse's actions demonstrate:

analgesic titration.
empathy.
independence.
patient advocacy.

9. Which is an accurate statement about the administration of acetaminophen (Tylenol) to children?

Acetaminophen (Tylenol) affects platelet aggregation.
Acetaminophen (Tylenol) causes gastric irritation.
Acetaminophen (Tylenol) does not have an analgesic ceiling.
Acetaminophen (Tylenol) has an analgesic ceiling.

10. Which behavioral therapy works best to relieve pain with muscle tension and spasms in patients who are anxious about their pain?

Distraction
Hypnosis
Relaxation
Stress management

11. The pain management nurse follows the recommended protocol for preventing constipation when starting a patient on opioids by:

adding bulk fiber to the diet.
giving the patient enemas as needed.
increasing fluids and exercise.
using a bowel stimulant and stool softener.

12. A 35-year-old male patient with testicular cancer is joking and playing cards with his roommate. When assessed by the pain management nurse, the patient rates his pain as a 7 on a numeric pain rating scale of 0 to 10. The nurse concludes that the patient's behavior:

is an emotional reaction to the anticipated pain.
is in anticipation of future pain.
is more indicative of the need for pain medication than the pain rating.
may be in conflict with the pain rating, and accepts the report of pain.

13. An older adult patient is discharged from the hospital with nortriptyline (Pamelor) for neuropathic pain. Which statement indicates the patient's need for additional education?

“I will chew sugarless gum and mints.”
“I will drink carbonated beverages.”
“I will take my medication at breakfast.”
“I will use a humidifier at bedtime.”

14. Which nonpharmacologic intervention is difficult to use with older adults who are cognitively impaired?

Aromatherapy
Distraction
Guided imagery
Heat application

15. An 85-year-old male patient, with a history of prostate cancer and metastasis to the lumbar spine, is receiving methadone (Dolophine), 10 mg, 3 times a day. The patient's wife tells the pain management nurse that her husband exhibits a lack of motivation, loss of appetite, and an inability to get out of bed. The nurse initially focuses on the patient's:

need for antidepressants.
pain assessment.
physical therapy evaluation.
psychological evaluation.

16. Which medication is the best choice to treat breakthrough pain for a patient who is currently receiving methadone (Dolophine), 10 mg, every 8 hours?

Methadone (Dolophine)
Immediate release morphine (MS IR)
Sustained release morphine (MS Contin)
Transdermal fentanyl (Duragesic)

17. A 75-year-old female patient comes to the oncology clinic for management of chronic cancer pain. The patient has been prescribed morphine sulfate (MS Contin), 30 mg, every 12 hours. The patient states that she is taking the medicine only when the pain becomes severe because of her husband's concern about addiction. The pain management nurse responds:

“It is okay to continue doing what you are doing.”
“The risk of developing addiction when taking opioids for pain is very low.”
“We need to consider other alternatives for managing your pain.”
“You must take the medication as prescribed, regardless of your husband's concerns.”

18. The pain management nurse assesses a male patient who has complex regional pain syndrome. The nurse is concerned about the patient's depressed mood because he has made comments that he "can't live with this pain." The nurse further assesses for suicide risk because:

decreased pain thresholds and suicidal thoughts are frequently seen in patients with complex regional pain syndrome.
suicidal thoughts are common in patients with chronic pain.
suicidal thoughts are often expressed by patients with acute pain.
verbalization of suicidal thoughts is a way for patients to get attention.

19. A focused examination for fibromyalgia includes assessment for:

autonomic changes, peripheral neuropathy, and skin fold tenderness.
morning fatigue, widespread body pain, and focal tenderness.
skin rashes, edema, and constipation.
unilateral weakness, hyperesthesia, and allodynia.

20. Biofeedback is a therapy used to:

develop psycho-physiologic self-regulation.
enhance drug delivery.
increase release of serotonin.
promote neuronal regeneration.

21. The pain management nurse notices a male patient grimacing as he moves from the bed to a chair. The patient tells the nurse that he is not experiencing any pain. The nurse's response is to:

clarify the patient's report by reviewing the patient's nonverbal behavior.
confronting the patient's denial of pain.
obtaining an order for pain medication.
supporting the patient's stoic behavior.

22. The pain management nurse assesses a 67-year-old patient for reports of episodic, sudden-onset, and right-sided facial pain. The patient describes the pain as fleeting, electric-like, and triggered by light touch and brushing of the teeth. The pain management nurse suspects:

facet syndrome.
myofascial pain syndrome.
temporomandibular disorder.
trigeminal neuralgia.

23. Which is the most effective method for teaching strengthening exercises to patients with chronic pain?

Distributing a local listing of strengthening exercise classes
Providing demonstration and return demonstration
Providing handouts with pictures
Viewing a self-instruction video

24. When teaching a 65-year-old patient to use a pain scale, a pain management nurse anticipates that:

additional time is needed for the patient to process the information.
older adults are unable to use pain scales reliably.
the Pain Assessment in Advanced Dementia Scale is appropriate for the patient.
the patient's family is included in the education sessions.

25. A 51-year-old female patient who has metastatic breast cancer is stabilized on oxycodone (OxyContin) for her pain. The patient exhausts her family medical leave act, is forced to leave her job, and, subsequently, loses her health insurance. What is the pain management nurse's best course of action for the patient?

Encouraging the patient to cut the pain tablets in half to make them last longer
Offering information to the patient about
pharmaceutical assistance programs
Preparing the patient to be weaned off of oxycodone (OxyContin)
Providing the patient with information on the nearest free clinic

Question 1

The correct answer is Larger doses of opioids are needed to control pain compared to several weeks earlier..

Question 2

The correct answer is home remedies used to treat pain..

Question 3

The correct answer is alcohol..

Question 4

The correct answer is allodynia..

Question 5

The correct answer is Location, quality, and onset.

Question 6

The correct answer is a decrease in renal function..

Question 7

The correct answer is unilaterally..

Question 8

The correct answer is patient advocacy..

Question 9

The correct answer is Acetaminophen (Tylenol) has an analgesic ceiling..

Question 10

The correct answer is Relaxation.

Question 11

The correct answer is using a bowel stimulant and stool softener..

Question 12

The correct answer is may be in conflict with the pain rating, and accepts the report of pain..

Question 13

The correct answer is “I will take my medication at breakfast.”.

Question 14

The correct answer is Guided imagery.

Question 15

The correct answer is pain assessment..

Question 16

The correct answer is Immediate release morphine (MS IR).

Question 17

The correct answer is “The risk of developing addiction when taking opioids for pain is very low.”.

Question 18

The correct answer is suicidal thoughts are common in patients with chronic pain.

Question 19

The correct answer is morning fatigue, widespread body pain, and focal tenderness.

Question 20

The correct answer is develop psycho-physiologic self-regulation.

Question 21

The correct answer is clarify the patient's report by reviewing the patient's nonverbal behavior.

Question 22

The correct answer is trigeminal neuralgia.

Question 23

The correct answer is Providing demonstration and return demonstration.

Question 24

The correct answer is additional time is needed for the patient to process the information.

Question 25

The correct answer is Offering information to the patient about pharmaceutical assistance programs.

Sample Questions - American Board of Pain Medicine

1. One of the effects created by activation or increased release of substance P is

(A) vasoconstriction.
(B) membrane stabilization.
(C) analgesia.
(D) vasodilation.

2. A 23-year-old female patient, who was recently discharged from the hospital following open reduction and internal fixation of a fractured femur, suddenly develops severe chest pain. Which of the following medications in her history would seem to be implicated in the etiology of her pain?

(A) Oral contraceptives
(B) Nonsteroidal anti-inflammatory agents
(C) Opioid analgesics
(D) Benzodiazepines

3. Patients who present with fever and pain of recent onset over the neck, upper back, chest, and upper limbs should be assessed for the possibility of abscess in the

(A) cervical epidural space.
(B) posterior nasopharynx.
(C) subdiaphragmatic space.
(D) T 7-8 disk space.

4. Disability due to chronic pain is felt to be primarily related to the

(A) number of somatic sites in which pain exists.
(B) reinforcement of pain behaviors.
(C) presence of a life-threatening disease.
(D) presence of neuropathic, as opposed to muscular, pain causes.

5. Further testing with CT scan or MRI is mandatory
in headaches accompanied by all of the following EXCEPT
(A) prolonged long-term, unchanging band-like headaches.
(B) hemiparesis and contralateral sensory deficit.
(C) the appearance of seizures.
(D) olfactory hallucinations.

6. Referral to a multidisciplinary pain center is usually most appropriate when patients demonstrate evidence of

(A) purely psychiatric mechanisms.
(B) purely neuropathic mechanisms.
(C) both psychological tension and physical muscle tension.
(D) both somatic and psychological factors.

7. Which tricyclic antidepressant is most appropriate for treatment of pain in an 80-year-old male with postherpetic neuralgia and urinary retention?

(A) Amitriptyline (Elavil)
(B) Doxepin (Sinequan)
(C) Desipramine (Norpramin)
(D) Imipramine (Tofranil)

8. In disability determination under most workers' compensation systems, the presence of pain is given

(A) more attention than the underlying physical impairment.
(B) as much attention as the underlying physical impairment.
(C) less attention than the underlying physical impairment.
(D) no attention whatsoever.

9. Which of the following is true regarding the use of antidepressants to reduce chronic pain?

(A) Only tertiary amine tricyclics are effective.
(B) Serotonergic agents are not clearly superior to noradrenergic ones.
(C) Serotonin potentiation is a necessary characteristic of effective agents.
(D) Only noradrenergic agents are effective.

10. DREZ lesions have been documented to provide long-term pain relief in

(A) cervical root avulsion.
(B) sciatica.
(C) diabetic neuropathy.
(D) thalamic pain syndrome.

11. A 52-year-old patient presents with a history of acute low back pain, without trauma, which is unrelieved by bed rest and is associated with paroxysms of pain and an elevated erythrocyte sedimentation rate. Radiographs of the spine reveal an absent pedicle. The most likely diagnosis is

(A) lupus erythematosus.
(B) multiple myeloma.
(C) metastatic lesion.
(D) disc space infection.

12. Which of the following is true regarding patients with cluster headaches?

(A) They are more likely to be female.
(B) They are likely to lie in a quiet, dark room with an ice pack over the affected temple during an attack.
(C) They are usually nonsmokers and nondrinkers.
(D) They are known to attempt suicide secondary to their pain.

13. Which of the following is true of the physical or sensory component of pain perception?

(A) It is less variable than the anxiety produced by the pain.
(B) It is more variable than the anxiety produced by the pain.
(C) It is generally equal to the anxiety produced by the pain.
(D) It is reduced in patients with hypochondriasis.

14. A patient who has been taking high doses of benzodiazepines and opioids experiences withdrawal symptoms during detoxification. Which of the following specifically indicates that the opioid is being tapered too rapidly?

(A) Hyperreflexia
(B) Diaphoresis
(C) Hyperactive bowel sounds
(D) Tachycardia

15. The depression commonly seen in those with chronic pain of nonmalignant origin differs from the most typical major depressions in that in the former there is likely to be

(A) anhedonia.
(B) weight gain.
(C) guilty ruminations.
(D) insomnia.

16. All of the following are true of migraine headache EXCEPT

(A) Aura (prodrome) is not present in common migraine.
(B) The neurologic symptoms of classic migraine may persist beyond the headache phase.
(C) Ergotamine (Ergostat) is effective in treating acute attacks when used daily for 7-14 days.
(D) 70% of migraine patients have a positive family migraine history.

17. The essential feature of pain that can be used to differentiate it from other somatic sensations is its

(A) intensity.
(B) threshold.
(C) chronicity.
(D) unpleasantness.

18. Aching pain in the suprapubic region is most likely caused by abnormalities of the
(A) ureter.
(B) prostate.
(C) coccyx.
(D) sacroiliac joints.
Answer Key:

1. D, 2. A, 3. A, 4. B, 5. A, 6. D, 7. C, 8. C, 9. B, 10. A, 11. C, 12. D, 13. A, 14. C, 15. B, 16. C, 17. D, 18. B

Q: Should I only take pain medicine when I have bad pain?

A: Pain is easier to control when it is mild. This may mean taking your pain medicine on a regular or "scheduled" basis. You should take your pain medicine when you start to notice pain. Waiting too long can make it much harder to control.

Q: What if my pain medicine makes me sick to my stomach?

A: Sometimes pain medicine can upset your stomach. There are medicines we can use to relieve this. Sometimes your pain medicine will be changed to make this better.

Q: Will my pain medicine make me constipated?

A: Constipation is a common side effect of pain medicine. You can still take your medicine for pain. Check with your doctor or nurse about using a laxative or stool softener to prevent constipation.

Q: What if my pain medicine makes me too sleepy?

A: Pain medicine can make you drowsy at first. Often this will go away after the first few doses. Check with your doctor if drowsiness lasts longer than a couple of days.

Q: Will I become addicted to pain medicine?

A: When pain medication is used on a short-term basis to control pain, you will not become addicted to it. When your pain is controlled, you will feel better, move better and your risk of complications such as pneumonia or blood clots is greatly reduced.
Knee Pain
Pain (Chronic Pain)
Pain in the Arm (Arm Pain)
Pain in the Butt (Buttock Pain)
Pain in the Heel (Heel Pain)
Pain in the Hip (Hip Pain)
Pain in the Jaw (Jaw Pain)
Pain, Back (Back Pain)
Pain, Breast (Breast Pain)
Pain, Cervical (Neck Pain)
Pain, Chest (Chest Pain)
Pain, Chest with Breathing (Pleurisy)
Pain, Dental (Toothache)
Pain, Ear (Earache)
Pain, Elbow (Elbow Pain)
Pain, Head (Headache)
Pain, Joint (Joint Pain)
Pain, Knee (Knee Pain)
Pain, Menstrual (Menstrual Cramps)
Pain, Neck (Neck Pain)
Pain, Tailbone (Coccydynia)
Pain, Throat (Sore Throat)
Pain, Tooth (Toothache)
Pain, Vagina (Vaginal Pain)
Painful Gums
Painful Urination (Burning Urination)
Pain with urination
Painful bowel movements
Painful ejaculation
Painful red lump on eyelid
Vaginal Pain
Managing Pain After Surgery

Dealing with post-surgery pain begins before your operation.

What should you know about pain killers after surgery?

Why is pain control so important?

In addition to keeping you comfortable, pain control can help you recover faster and may reduce your risk of developing certain complications after surgery, such as pneumonia and blood clots. If your pain is well controlled, you will be better able to complete important tasks such as walking and deep breathing exercises.

What kinds of pain will I feel after surgery?

You may be surprised at where you experience pain after surgery. Often times the incision itself is not the only area of discomfort. You may or may not feel the following:

Muscle pain - You may feel muscle pain in the neck, shoulders, back, or chest from lying on the operating table.

Throat pain - Your throat may feel sore or scratchy. Movement pain - Sitting up, walking, and coughing are all important activities after surgery, but they may cause increased pain at or around the incision site. What can I do to help keep my pain under control? Important!: Your doctors and nurses want and need to know about pain that is not adequately controlled. If you are having pain, please tell someone! Don’t worry about being a “bother.”

You can help the doctors and nurses "measure" your pain. While you are recovering, your doctors and nurses will frequently ask you to rate your pain on a scale of 0 to 10, with "0" being "no pain" and "10" being "the worst pain you can imagine." Reporting your pain as a number helps the doctors and nurses know how well your treatment is working and whether to make any changes. Keep in mind that your comfort level (ie, ability to breathe deeply or cough) is more important than absolute numbers (ie, pain score).

Who is going to help manage my pain?

You and your surgeon will decide what type of pain control would be most acceptable for you after surgery. Your surgeon may choose to consult the Acute Pain Management Service to help manage your pain following surgery. Doctors on this service are specifically trained in the types of pain control options described on the next several pages.

You are the one who ultimately decides which pain control option is most acceptable. The manager of your post-surgical pain -- your surgeon or the Acute Pain Management Service doctor -- will review your medical and surgical history, check the results from your laboratory tests and physical exam, then advise you about which pain management option may be best suited to safely minimize your discomfort.

After surgery, you will be assessed frequently to ensure that you are comfortable and safe. When necessary, adjustments or changes to your pain management regimen will be made.

Types of Pain-Control Treatments

You may receive more than one type of pain treatment, depending on your needs and the type of surgery you are having. All of these treatments are relatively safe, but like any therapy, they are not completely free of risk. Dangerous side effects are rare. Nausea, vomiting, itching, and drowsiness can occur. These side effects are usually easily treated in most cases.

Intravenous Patient-Controlled Analgesia (PCA)

Patient-controlled analgesia (PCA) is a computerized pump that safely permits you to push a button and deliver small amounts of pain medicine into your intravenous (IV) line, usually in your arm. No needles are injected into your muscle. PCA provides stable pain relief in most situations. Many patients like the sense of control they have over their pain management.

The PCA pump is programmed to give a certain amount of medication when you press the button. It will only allow you to have so much medication, no matter how often you press the button, so there is little worry that you will give yourself too much.

One way that you may get too much medication from the PCA pump is if a family member presses the PCA button for you. This removes the patient control aspect of the therapy, which is a major safety feature. Do not allow family members or friends to push your PCA pump button for you. You need to be awake enough to know that you need pain medication.

Patient-Controlled Epidural Analgesia

Many people are familiar with epidural anesthesia because it is frequently used to control pain during childbirth. Patient-Controlled Epidural Analgesia uses a PCA pump to deliver pain-control medicine into an epidural catheter (a very thin plastic tube) that is placed into your back.

Placing the epidural catheter (to which the PCA pump is attached) usually causes no more discomfort than having an IV started. A sedating medication, given through your IV, will help you relax. The skin of your back will be cleaned with a sterile solution and numbed with a local anesthetic. Next, a thin needle will be carefully inserted into an area called the "epidural space." A thin catheter will be inserted through this needle into the epidural space, and the needle will then be removed. During and after your surgery, pain medications will be infused through this epidural catheter with the goal of providing you with excellent pain control when you awaken. If additional pain medication is required, you can press the PCA button.

Epidural analgesia is usually more effective in relieving pain than intravenous medication. Patients who receive epidural analgesia typically have less pain when they take deep breaths, cough, and walk, and they may recover more quickly. For patients with medical problems such as heart or lung disease, epidural analgesia may reduce the risk of serious complications such as heart attack and pneumonia.

Epidural analgesia is safe, but like any procedure or therapy, not risk free. Sometimes the epidural does not adequately control pain. In this situation, you will be given alternative treatments or be offered replacement of the epidural. Nausea, vomiting, itching, and drowsiness can occur. Occasionally, numbness and weakness of the legs can occur, which disappears after the medication is reduced or stopped. Headache can occur, but this is rare. Severe complications, such as nerve damage and infection, are extremely rare.

Nerve Blocks

You may be offered a nerve block to control your pain after surgery. Whereas an epidural controls pain over a broad area of your body, a nerve block is used when pain from surgery affects a smaller region of your body, such as an arm or leg. Sometimes a catheter similar to an epidural catheter is placed for prolonged pain control. There are several potential advantages of a nerve block. It may allow for a significant reduction in the amount of opioid (narcotic) medication, which may result in fewer side effects, such as nausea, vomiting, itching, and drowsiness.

In some cases, a nerve block can be used as the main anesthetic for your surgery. In this case, you will be given medications during your surgery to keep you sleepy, relaxed, and comfortable. This type of anesthesia provides the added benefit of pain relief both during and after your surgery. It may reduce your risk of nausea and vomiting after surgery. You, your anesthesiologist, and your surgeon will decide before surgery if a nerve block is a suitable pain management or anesthetic option for you.

Pain Medications Taken by Mouth

At some point during your recovery from surgery, your doctor will order pain medications to be taken by mouth (oral pain medications). These may be ordered to come at a specified time, or you may need to ask your nurse to bring them to you. Make sure you know if you need to ask for the medication! Most oral pain medications can be taken every 4 hours.

Important!: Do not wait until your pain is severe before you ask for pain medications. Also, if the pain medication has not significantly helped within 30 minutes, notify your nurse. Extra pain medication is available for you to take. You do not have to wait 4 hours to receive more medication.

What are some of the risks and benefits associated with pain medication?

Opioids (Narcotics) after surgery: medications such as morphine, fentanyl, hydromorphone Benefits: Strong pain relievers. Many options are available if one is causing significant side effects.

Risks: May cause nausea, vomiting, itching, drowsiness, and/or constipation. The risk of becoming addicted is extremely rare.

Opioids (Narcotics) at home (Percocet®, Vicodin®, Darvocet®, Tylenol ®)

Benefits: Effective for moderate to severe pain. Many options available.

Risks: Nausea, vomiting, itching, drowsiness, and/or constipation. Stomach upset can be lessened if the drug is taken with food. You should not drive or operate machinery while taking these medications. Note: These medications often contain acetaminophen (Tylenol®). Make sure that other medications that you are taking do not contain acetaminophen, because too much of it may damage your liver.

Non-Opioid (Non-narcotic) Analgesics (Tylenol®, Feverall®)

Benefits: Effective for mild to moderate pain. They have very few side effects and are safe for most patients. They often decrease the requirement for stronger medications, which may reduce the incidence of side effects.

Risks: Liver damage may result if more than the recommended daily dose is used. Patients with pre-existing liver disease or those who drink significant quantities of alcohol may be at increased risk.

Nonsteroidal Anti-inflammatory Drugs (NSAIDS) ibuprofen (Advil®), naproxen sodium (Aleve®), celecoxib (Celebrex®)

Benefits: These drugs reduce swelling and inflammation and relieve mild to moderate pain. Ibuprofen and naproxen sodium are available without a prescription, but you should ask your doctor about taking them. They may reduce the amount of opioid analgesic you need, possibly reducing side effects such as nausea, vomiting, and drowsiness. If taken alone, there are no restrictions on driving or operating machinery.

Risks: The most common side effects of nonsteroidal anti-inflammatory medication (NSAIDS) are stomach upset and dizziness. You should not take these drugs without your doctor's approval if you have kidney problems, a history of stomach ulcers, heart failure or are on "blood thinner" medications such as Coumadin® (warfarin), Lovenox® injections, or Plavix®.

Are there ways I can relieve pain without medication? Yes, there are other ways to relieve pain and it is important to keep an open mind about these techniques. When used along with medication, these techniques can dramatically reduce pain.

Relaxation tapes or Guided Imagery is a proven form of focused relaxation that coaches you in creating calm, peaceful images in your mind -- a "mental escape." For the best results, practice using the tape or CD before your surgery, and then use it twice daily during your recovery. __________ has had great success with this relaxation program. You can get relaxation tapes at a bookstore, or rent CDs from your library. Guided Imagery tapes and CDs are available for _______ at the __________. Finally, you can bring a battery-operated listening device to the hospital to play prior to surgery and during your hospital stay.

Listening to soft music, changing your position in bed, or tuning in to channel 36 (Cleveland Clinic’s in-hospital relaxation channel) are additional methods to relieve or lessen pain.

At home, heat or cold therapy may be an option that your surgeon may choose to help reduce swelling and control your pain. Specific instructions for the use of these therapies will be discussed with you by your surgical team.

If you have an abdominal or chest incision, you will want to splint the area with a pillow when you are coughing or breathing deeply to decrease motion near your incision. You will be given a pillow in the hospital. Continue to use it at home as well.

Lastly, make sure you are comfortable with your treatment plan. Talk to your doctor and nurses about your concerns and needs. This will help avoid miscommunication, stress, anxiety, and disappointment, which may make pain worse. Keep asking questions until you have satisfactory answers. You are the one who will benefit.

How can I control pain at home? v You may be given prescriptions for pain medication to take at home. These may or may not be the same pain medications you took in the hospital. Talk with your doctor about which pain medications will be prescribed at discharge.

Note: Make sure your doctor knows about pain medications that have caused you problems in the past. This will prevent possible delays in your discharge from the hospital.

Preparation for Your Discharge:

Your doctors may have already given you your prescription for pain medication prior to your surgery date. If this is the case, it is best to be prepared and have your medication filled and ready for you when you come home from the hospital.

If you do not receive your prescription for pain medication until after the surgery, make sure a family member takes your prescription and either gets it filled at one of the ________ Pharmacies on campus or soon after your discharge from the hospital. It is important that you ARE PREPARED in case you have pain.

Make sure you wear comfortable clothes, and keep your coughing and deep breathing pillow with you.

You may want to have your relaxation music available for your travels.

If you are traveling by plane, make sure you have your pain pills in your carry-on luggage in case the airline misplaces your checked luggage.

While at Home:

Remember to take your pain medication before activity and at bedtime. Your doctor may advise you to take your pain medication at regular intervals (such as every 4 to 6 hours).

Be sure to get enough rest. If you are having trouble sleeping, talk to your doctor.

Use pillows to support you when you sleep and when you do your coughing and deep breathing exercises.

Try using the alternative methods discussed earlier. Heating pads or cold therapy, guided imagery tapes, listening to soft music, changing your position in bed, and massage can help relieve your pain.

NOTE: If you need to have stitches or staples removed and you are still taking pain medications, be sure to have a friend or family member drive you to your Clinic appointment. Commonly, you should not drive or operate equipment if you are taking opioid (narcotic)-containing pain medications. Check the label of your prescription for any warnings or ask your doctor, nurse, or pharmacist.

Frequently asked questions

I am nervous about getting “hooked” on pain pills. How do I avoid this?

The risk of becoming addicted to pain medication after surgery is very small. The bigger risk is a possible prolonged recovery if you avoid your pain medications, and cannot effectively do your required activities. If you are concerned about addiction, or have a history of substance abuse (alcohol or any drug), talk with your doctors. They will monitor you closely during your recovery. If issues arise following surgery, they will consult the appropriate specialists.

I am a small person who is easily affected by medicine. I am nervous that a “normal” dose of pain medication will be too much for me. What should I do? During recovery, your healthcare team will observe how you respond to pain medication and make changes as needed. Be sure to communicate with your doctors any concerns you have prior to surgery. The relatively small doses of pain medication given after surgery are highly unlikely to have an exaggerated effect based on your body size.

I don't have a high tolerance for pain. I am afraid that the pain will be too much for me to handle. What can I do?

Concern about pain from surgery is very normal. The most important thing you can do is to talk with your surgeon and anesthesiologist about your particular situation. Setting pain control goals with your doctors before surgery will help them better tailor your pain treatment plan. Treating pain early is easier than treating it after it has set in. If you have had prior experiences with surgery and pain control, let your doctor know what worked or what did not work. Remember, there are usually many options available to you for pain control after surgery.

I normally take Tylenol® if I get a headache. Can I still take Tylenol® for a headache if I am on other pain medication?

As discussed earlier, before taking any other medication, be sure to talk to your doctor. Some of the medications prescribed for use at home contain acetaminophen (Tylenol®) and if too much is taken, you may become ill. In order to avoid getting too much of any medication, discuss this issue with your doctor BEFORE you leave the hospital.

Play an Active Role in Your Pain Control Ask your doctors and nurses about: Pain and pain control treatments and what you can expect from them. You have a right to the best level of pain relief that can be safely provided. Your schedule for pain medicines in the hospital. How you can participate in a pain-control plan. Inform your doctors and nurses about: Any surgical pain you have had in the past. How you relieved your pain before you came to the hospital.

Pain you have had recently or currently. Pain medications you have taken in the past and cannot tolerate.

Pain medications you have been taking prior to surgery Any pain that is not controlled with your current pain medications.

You should:

Help the doctors and nurses “measure” your pain and expect staff to ask about pain relief often and to respond quickly when you do report pain. Ask for pain medicines as soon as pain begins. Tell us how well your pain is relieved and your pain relief expectations.

Use other comfort measures for pain control -- listening to relaxation or soft music, repositioning in bed, etc.

Toothache
What is Toothache?
What is Fluoride?
How Do I Brush My Teeth?
When Do Teeth Start to Come Through?
How Do I Know Which Toothbrush To Choose?
What causes a toothache?
When Should I See a Dentist About a Toothache?
What Happens When I Go to the Dentist for a Toothache?
What Treatments Are Available for a Toothache?
How Can Toothaches Be Prevented?
What is Toothache?
Toothache can be painful and upsetting for adults, but can be especially so for children.

What is Fluoride?
Fluoride is one of the best ways to help prevent against tooth decay.

This is because fluoride is a naturally occurring mineral which combines with the tooth's enamel to strengthen it!

How Do I Brush My Teeth?
Cleaning your teeth is really important and should be part of everyone's daily hygiene routine. Being taught how to brush effectively - including those hard to reach areas - will set you up for life and will help spare you tooth decay, fillings and, of course, bad breath! Teeth should be brushed for at least two minutes twice a day; once just before bedtime, and at least once again at another time during the day, usually in the morning.

Supervised Brushing – Infants

If you are very young your Mum or Dad will brush your teeth for you; usually by starting to brush their baby's gums with a soft toothbrush at bath time, or even let their baby have a go themselves as long as they are supervised. This technique establishes tooth brushing as part of a daily routine. Mum or Dad will brush a baby's teeth with fluoride toothpaste as soon as the first milk tooth breaks through (usually at around six months, but it can be earlier or later). It's important to use a fluoride paste from infancy as it helps prevent and control tooth decay.

Supervised Brushing - Toddlers

Children under the age of three can use a smear of family toothpaste containing fluoride. Mum or Dad will still need to brush your teeth for you, and may find it easier to stand or sit behind you at the sink, cradling your chin in their hand so that they can reach your top and bottom teeth more easily.

Some Top Tips that your Mum or Dad may find useful are:-
•When the first teeth start to come through, try using a children's toothbrush with a small smear of toothpaste.
•It is important to supervise your child's brushing until they are at least seven.
•Once all the teeth have come through, use a small-headed soft toothbrush in small circular movements and try to concentrate on one section at a time.
•Don't forget to brush gently behind the teeth and onto the gums.
•If possible make tooth brushing a routine - preferably in the morning, and last thing before your child goes to bed.
•Remember to encourage your child, as praise will often get results!

Mum or Dad should make sure that all the toothpaste is spat out, and not swallowed, but also not to rinse your mouth with lots of water as this washes all of the fluoride away.

Supervised Brushing – Under 7’s

When you are a bit older, but still under seven years old, you can try to brush your own teeth, but, you should always be supervised during tooth brushing, either by helping to brush your teeth, or by watching how you do it. When brushing your teeth, always start and finish at the same point. That way you know when you've done the job properly. With a pea sized blob of fluoride toothpaste on the brush, start with the top set back teeth and make small circular motions with the brush, moving gradually around to the other side of the mouth. When you get to the last tooth, bring the brush round and do the same with the inside of the teeth. This is often the part that people forget, but it's also likely to be where bacteria and tartar (calculus - the hard stuff that dentists scrape off teeth) will occur, so make sure you brush properly! Then do exactly the same with the bottom set. Children between the ages of three and six should use a pea-sized blob of toothpaste containing fluoride; don’t eat or lick the toothpaste from the tube!

Unsupervised Brushing – 7 Years +

When you reach seven or eight years old you should be able to brush your own teeth without Mum or Dads help. However, you must always follow the technique described on this page to help remind you how to clean your teeth properly.

When Do Teeth Start to Come Through?
Your first teeth will start to come through any time between six months and a year old. These teeth, also known as "milk", "primary" or "baby" teeth, should all be through by the age of three. By the time you are six, you should start to lose your "milk" teeth and get your permanent teeth. By fourteen, you should have 28 out of your 32 permanent teeth. The final four, the "wisdom" teeth, tend to come through between the ages of 18 and 25.

Milk Teeth

When you are born, almost all of your milk teeth (also known as primary or baby teeth) already have formed, but are still hidden in the gums. These teeth usually begin to come through the gums (erupt) at about 6 months of age; some babies get teeth earlier, and some get them later, and this is normal. Usually, the two front bottom teeth come through first. Next, the four front top teeth and two more bottom teeth, after that teeth slowly begin to fill the mouth. Teeth usually come in two at a time, one on either side of the jaw. You should have all 20 primary teeth by the time you are 3 years old. These teeth usually have spaces between them; the spaces help to make sure that there is enough room for the permanent teeth when they are ready to erupt.

Permanent Teeth

Usually you would start to lose your "milk" teeth and replace them with adult teeth when you are 6 or 7 years old. Some children start losing teeth earlier, and others start later, and again this is normal. The older that you are when your primary teeth come through is more important than when they start to come through. Usually the first permanent teeth to come thorugh are the lower front four teeth. However, some children get their first permanent molars (sometimes called the 6-year molars) first. 6-year molars come in behind the primary teeth, but they do not replace primary teeth. Around 11 or 12 years old, the second permanent molars (also called 12-year molars) come in behind the 6-year molars. By the time you are 13 years old, most of your permanent teeth will be in place. Wisdom teeth - or third molars - come in between the ages of 17 and 21. However, some people don't get any wisdom teeth, or don't get all four. More often, wisdom teeth develop, but there may not be room in the mouth for them and eventually they may have to be removed.

How Do I Know Which Toothbrush To Choose?
There are many different types of children's toothbrushes available, including brightly coloured brushes, those with favourite characters on the handles, and even some with timers!

Choose one that you like and it will encourage you to brush your teeth!

The most important thing is to make sure that you have a small-headed toothbrush with soft, nylon bristles, suitable for your age.

What causes a toothache?

When Should I See a Dentist About a Toothache?
What Happens When I Go to the Dentist for a Toothache?
What Treatments Are Available for a Toothache?
How Can Toothaches Be Prevented?
A toothache is a pain in or around a tooth that may be caused by:
•Tooth decay
•Abscessed tooth
•Tooth fracture
•A damaged filling
•Repetitive motions, such as chewing gum or grinding teeth
•Infected gums
•Infection
•Gum disease
•Grinding teeth (bruxism)
•Tooth trauma
•An abnormal bite
•Tooth eruption (in babies and school-age children)

TMJ/TMD (Temporomandibular Joint Disorder), sinus or ear infections, and tension in the facial muscles can cause discomfort that resembles a toothache, but often these health problems are accompanied by a headache.

Pain around the teeth and the jaws can be symptoms of heart disease such as angina. If your dentist suspects a medical illness could be the cause of your toothache, he or she may refer you to a physician.

If you have a toothache, you may have a cavity or advanced gum disease. The first sign of decay may be the pain you feel when you eat something sweet, very cold or very hot. If the pulp – the inside of the tooth that has tissue and nerves – has become irritated, this can cause pain in your tooth.

What symptoms accompany a toothache?
Because the symptoms of a toothache may resemble other medical conditions or dental problems, it can be difficult to diagnose the cause without a complete evaluation by your dentist. If you notice pus near the source of the pain, your tooth may have become abscessed, causing the surrounding bone to become infected. Or the pus could indicate gum disease, which is usually characterized by inflammation of the soft tissue, bleeding gums and abnormal loss of bone surrounding the teeth.

Contact your dentist immediately if you have any of the following symptoms:

•Fever
•Difficulty breathing or swallowing
•Swelling around the tooth area
•Pain when you bite
•A foul-tasting discharge
•Continuous lasting pain

Symptoms of a toothache may include:

Recommended Related to Oral HealthDental Tips for Looking Younger

Your mouth is more than just a pretty smile. It's also a gateway to your overall health. Keeping that gateway clean may keep you healthier longer -- and looking younger. “Just as white, straight teeth convey youth, a smile with crooked, discolored, worn, or missing teeth is associated with an aged look. “The adage ‘long in the tooth,’ used to describe older persons, reflects the fact that gum disease causes.

•Tooth pain that may be sharp, throbbing, or constant. In some people, pain results only when pressure is applied to the tooth.
•Swelling around the tooth
•Fever or headache
•Foul-tasting drainage from the infected tooth
Continue reading below.

When Should I See a Dentist About a Toothache?
See your dentist as soon as possible about your toothache if:
•You have a toothache that lasts longer than 1 or 2 days
•Your toothache is severe
•You have a fever, earache, or pain upon opening your mouth wide Proper identification and treatment of dental infections is important to prevent its spread to other parts of the face and skull and possibly even to the bloodstream.

What Happens When I Go to the Dentist for a Toothache?
To treat your toothache, your dentist will first obtain your medical history and conduct a physical exam. He or she will ask you questions about the pain, such as when the pain started, how severe it is, where the pain is located, what makes the pain worse, and what makes it better. Your dentist will examine your mouth, teeth, gums, jaws, tongue, throat, sinuses, ears, nose, and neck. X-rays may be taken as well as other tests, depending on what your dentist suspects is causing your toothache.

What Treatments Are Available for a Toothache?
Treatment for a toothache depends on the cause. If a cavity is causing the toothache, your dentist will fill the cavity or possibly extract the tooth, if necessary. A root canal might be needed if the cause of the toothache is determined to be an infection of the tooth's nerve. Bacteria that have worked their way into the inner aspects of the tooth cause such an infection. An antibiotic may be prescribed if there is fever or swelling of the jaw. Phototherapy with a cold laser may be used to reduce the pain and inflammation associated with the toothache.

How do I alleviate the pain if I cannot see my dentist right away?
Anyone with a toothache should see a dentist at once for diagnosis and treatment. If left untreated, a toothache and the condition that is causing it can worsen. However, if you are unable to schedule an emergency appointment, the following self-care treatment can temporarily alleviate pain and inflammation from a toothache:

•Rinse with warm salt water
•Gently floss teeth to dislodge any food particles trapped between teeth. •Take an over-the-counter pain reliever such as ibuprofen (Advil®) or acetaminophen (Tylenol®) to relieve pain.
•Never put aspirin or any other painkiller against the gums near the aching tooth, as it may burn the gum tissue.
•Apply an over-the-counter antiseptic containing benzocaine directly to the irritated tooth and gum to temporarily relieve pain. Direct application of oil of cloves (eugenol) may also help to numb the gums. The oil may be rubbed directly on the sore area, or soak a small piece of cotton and apply it to the sore tooth.
•If there has been some trauma to the tooth, a cold compress may be applied on the outside cheek to relieve pain or swelling.

How can my dentist help?
Your dentist will conduct a complete oral examination to determine the location and cause of the toothache, looking for signs of swelling, redness and obvious tooth damage. He or she may also take x-rays to look for evidence of tooth decay between teeth, a cracked or impacted tooth or a disorder of the underlying bone.

Your dentist also may prescribe pain medication or antibiotics to speed the healing of your toothache. If by the time you see your dentist your tooth has become infected, then treatment could require removal of the tooth or a root canal procedure, which involves removing the damaged nerve tissue from the middle of a tooth.

Is there a way to prevent a toothache?
The key to preventing toothaches is establishing a regular oral hygiene routine and sticking to it. For example, failure to brush and floss regularly after meals can significantly increase your risk of developing cavities, which can cause toothaches.

Here are a few tips to help reduce your risk for developing a toothache:

•Brush at least twice a day, preferably after meals and snacks.
•Floss at least once a day to prevent gum disease.
•Visit your dentist regularly for oral examinations and a professional cleaning.

Since most toothaches are the result of tooth decay, following good oral hygiene practices can prevent toothaches. Good oral hygiene practices consist of brushing regularly with a fluoride-containing toothpaste, flossing once daily, and seeing your dentist twice a year for professional cleaning. In addition to these practices, eat foods low in sugar and ask your dentist about sealants and fluoride applications.

How to Get Rid of Tooth Pain

Take pain medication. Use an over-the-counter pain reliever like aspirin, acetaminophen, ibuprofen, or naproxen sodium. Follow the instructions on the label to determine the correct dosage.
•For most pain relievers, you will take one or two pills every four to six hours. Exact dosages vary by medication and brand, however.

Orajel

Toothache Pain Relief
Teething Relief
Mouth & Canker Sore Pain Relief

Temporarily relieves pain caused by canker sores, cold sores, fever blisters, minor irritation or injury of the mouth and gums.
Active Ingredients: Benzocaine - 20.0 % (Oral Pain Reliever), Benzalkonium Chloride - 0.02 % (Oral Antiseptic), Zinc Chloride - 0.1 % (Oral Astringent) Inactive Ingredients: Allantoin, Carbomer, Edetate Disodium, Mentha Piperita (Peppermint) Oil, Polyethylene Glycol, Polysorbate 60, Propylene Glycol, Propyl Gallate, Purified Water, PVP, Sodium Saccharin, Sorbic Acid, Stearyl Alcohol

Rinse with salt water. Salt can strip the mouth of bacteria and draw moisture out from an inflamed gum surrounding a pained tooth, thereby soothing the area.
•Combine 1 tsp (5 ml) of salt with 8 oz (250 ml) of warm water.
•Rinse with this solution for 30 seconds before spitting it out. Repeat as needed.

Use a topical pain medication. Apply an over-the-counter topical ointment containing benzocaine directly to the tooth or gum experiencing pain. Follow the label directions to determine the proper amount and application.

Medication relevant to cause of pain (analgesics/pain relievers).
Pain Relievers: Understanding Your OTC Options
Opiate Pain Relievers for Chronic Pain

Pain Relievers: Understanding Your OTC Options

What types of OTC pain relievers are available?
Over-the-counter (OTC) pain relievers are medicines that you can buy without a prescription from your doctor. Two main types of OTC pain relievers are available. One type is acetaminophen (brand name: Tylenol). The second type is nonsteroidal anti-inflammatory drugs (also called NSAIDs). NSAIDs include the following:

•Aspirin (2 brand names: Bayer, St. Joseph)
•Ibuprofen (2 brand names: Advil, Motrin)
•Naproxen (brand name: Aleve)

Some products contain both acetaminophen and aspirin (brand names: Excedrin Extra Strength, Excedrin Migraine, Vanquish).

How do pain relievers work?
Acetaminophen seems to relieve pain and reduce fever by working on the parts of the brain that receive pain messages and control the body’s temperature.

NSAIDs relieve pain and fever by reducing the level of hormone-like substances (called prostaglandins) that your body makes. These substances cause the feeling of pain by irritating your nerve endings. They also are part of the system that helps your body control its temperature.

What types of problems can OTC pain relievers help?
Acetaminophen and NSAIDs help to reduce fever and relieve pain caused by headaches, muscle aches, and stiffness. NSAIDs can also reduce inflammation (swelling). Acetaminophen does not reduce inflammation.

OTC pain relievers can be helpful in treating many types of pain, such as pain from arthritis, earaches, back pain, and pain after surgery. They can also treat pain from the flu (influenza) or a cold, sinusitis, strep throat, or a sore throat. Children who may have the flu or chickenpox should not take aspirin because they have a higher risk for developing a condition called Reye syndrome.

Acetaminophen can be a good choice for relieving headaches and other common aches and pains. It can be used safely on a long-term basis by most people for arthritis and other chronic painful conditions.

Ibuprofen is helpful for relieving menstrual cramps and pain from inflammation (such as muscle sprains). If ibuprofen doesn’t work for you, naproxen may be an option.

Will an OTC pain reliever work as well as a prescription one?
For most people, OTC medicines are all that is needed to relieve pain or reduce fever. However, if an OTC medicine doesn’t help your pain or fever, it may be a sign you have a more serious problem or need a prescription medicine.

How do I safely take OTC pain relievers?
Read the directions on the drug facts label to learn how much medicine to take and how often to take it. If you have any questions about how much medicine to take, call your family doctor or pharmacist. Keep a record (1-page PDF; About PDFs) of which OTC medicines you are using and when you take them. If you need to go to the doctor, take this list with you.

Follow these tips to make sure you are taking the right amount of medicine:

•Take only the amount recommended on the medicine’s label. Don’t assume that more medicine will work better or quicker. Taking more than the recommended amount can be dangerous.
•If you are taking a prescription medicine, ask your doctor if it’s okay to also take an OTC pain reliever.
•Don’t use more than 1 OTC pain reliever at a time unless your doctor says it’s okay. They may have similar active ingredients that add up to be too much medicine.
•Check whether other OTC medicines you are taking contain acetaminophen. For example, many combination medicines for cold symptoms contain this active ingredient. Taking too much acetaminophen can cause health problems.

How can I safely store OTC pain relievers?
Store all medicines up and away, out of reach and sight of young children. Keeping medicines in a cool, dry place will help prevent them from becoming less effective before their expiration dates. Do not store medicines in bathrooms or bathroom cabinets, which are often hot and humid.

What are some common side effects of OTC pain relievers?
Side effects from OTC pain relievers are uncommon for healthy adults who only use pain relievers once in a while. If you have health problems or use OTC pain relievers often, talk with your family doctor.

Side effects with acetaminophen are rare. However, liver damage can occur if you take too much acetaminophen, or drink alcohol and take acetaminophen. For an adult, more than 3 grams of acetaminophen (6 extra-strength 500 mg tablets) a day is too much.

NSAIDs may cause upset stomach. They can also cause increased bruising or risk of bleeding in the stomach. When taken regularly, they may cause kidney damage. NSAIDs may also make high blood pressure worse.

Who shouldn’t take acetaminophen?
Do not take acetaminophen if you:
•Have severe kidney or liver disease
•Have 3 or more drinks that contain alcohol every day
•Are already taking another product containing acetaminophen

A note about infant acetaminophen
Starting in the summer of 2011, all manufacturers of infant acetaminophen, including Tylenol, changed the concentration (mg per mL) of acetaminophen in their medicines. This means that the instructions for giving these medicines to your child also have changed. The medicine you buy in the store may give different instructions than the medicine that you have at home in your medicine cabinet. Be sure to carefully read and follow the directions on the box or bottle of the product you are using. If you have questions, ask your family doctor or pharmacist. Learn more.

Who shouldn’t take NSAIDs?
Talk with your doctor before you take an NSAID, especially aspirin, if you:

•Are allergic to aspirin or other pain relievers
•Have 3 or more drinks that contain alcohol every day
•Have bleeding in the stomach or intestines, or have peptic (stomach) ulcers
•Have liver or kidney disease
•Have heart disease
•Take blood-thinning medicine or have a bleeding disorder
Children and teenagers younger than 18 years of age who may have the flu or chickenpox should not take aspirin because of the risk of Reye syndrome, which is a serious illness that can lead to death.

Can OTC pain relievers cause problems with any other medicines I take?
NSAIDs can interact with blood pressure medicines. Someone who takes medicine for high blood pressure and also takes an NSAID may find that the blood pressure medicine does not work as well as it should.

Opiate Pain Relievers for Chronic Pain
Chronic Pain | Chronic Pain Medicines
Generic Name Brand Name
fentanyl Duragesic
hydrocodone Norco, Vicodin
hydromorphone Dilaudid, Exalgo
morphine Astramorph, Avinza
oxycodone OxyContin, Percocet
Opioids are available in pills, liquids, or suckers to take by mouth, and in shot, skin patch, and suppository form.
How is chronic pain treated?

Treatment of chronic pain usually involves medicines and therapy. Medicines used for chronic pain include pain relievers, antidepressants and anticonvulsants. Different types of medicines help people who have different types of pain. You usually use long-acting medicines for constant pain. Short-acting medicines treat pain that comes and goes.

What drugs can treat chronic pain?
Many medicines can decrease pain, including the ones listed below. Each one may have side effects. Some side effects can be serious. It's important to listen to your family doctor carefully when he or she tells you how to use your pain medicine. If you have questions about side effects or about how much medicine to take, ask your doctor or your pharmacist.

Acetaminophen

Acetaminophen (one brand name: Tylenol) helps many kinds of chronic pain. Remember, many over-the-counter and prescription pain medicines have acetaminophen in them. If you're not careful, you could take more acetaminophen than is good for you. Taking too much acetaminophen could cause liver damage, especially if you drink alcohol. If you often find that you have to take more than 2 acetaminophen pills a day, tell your doctor.

Nonsteroidal Anti-Inflammatory Drugs (NSAIDs)

Other drugs that help with pain are called nonsteroidal anti-inflammatory drugs, or NSAIDs. Examples include aspirin, ibuprofen (two brand names: Motrin, Advil) and naproxen (one brand name: Aleve). NSAIDs come in both over-the-counter and prescription forms. These medicines can be taken just when you need them, or they can be taken every day. When these medicines are taken regularly, they build up in the blood to levels that fight the pain of inflammation (swelling) and also give general pain relief.

If your doctor wants you to take an NSAID, always take it with food or milk because the most common side effects are related to the stomach. They can also cause increased bruising or risk of bleeding in the stomach. When taken for long periods of time, they may cause kidney damage. NSAIDs may make high blood pressure worse or interfere with blood pressure medicines.

If you are taking other pain medicines, don't take NSAIDs without talking to your doctor first.

Narcotics

Narcotics can be addictive, so your family doctor will be careful about prescribing them. For many people who have severe chronic pain, these drugs are an important part of their therapy. If your doctor prescribes narcotics for your pain, be sure to carefully follow his or her directions. Tell your doctor if you are uncomfortable with the changes that may go along with taking these medicines, such as the inability to concentrate or think clearly. Do not drive or operate heavy machinery when taking these medicines.
When you're taking narcotics, it's important to remember that there is a difference between "physical dependence" and "psychological addiction." Physical dependence on a medicine means that your body gets used to that medicine and needs it in order to work properly. When you don't have to take the pain medicine any longer, your doctor can help you slowly and safely decrease the amount of medicine until your body no longer "needs" it.

Psychological addiction is the desire to use a drug whether or not it's needed to relieve pain. Using a narcotic this way can be dangerous and may not help your pain. If you have a psychological addiction to a narcotic, your doctor may give you another drug to help with your psychological problems. Or your doctor might recommend that you talk to a counselor. Your doctor might also change the medicine that you are addicted to by lowering the dose, changing to another drug or stopping the medicine altogether.

Narcotic drugs often cause constipation (difficulty having bowel movements). If you are taking a narcotic medicine, it's important to drink at least 6 to 8 glasses of water every day. Try to eat 2 to 4 servings of fresh fruits and 3 to 5 servings of vegetables every day. Be sure to tell your doctor if constipation becomes a problem for you. He or she may suggest taking laxatives to treat or prevent it.

Other medicines

Many drugs that are used to treat other illnesses can also treat pain. For example, carbamazepine is a seizure medicine that can also treat some kinds of pain. Amitriptyline is an antidepressant that can also help with chronic pain. Your doctor may want you to try one of these medicines to help control your pain. It can take several weeks before these medicines begin to work well.

Remember -- if you are taking any pain medicine, be sure to ask your doctor or pharmacist before you take any other medicine, either prescription or over-the-counter. You should also check with your doctor or pharmacist before taking an herbal supplement or vitamin.

Pelvic Pain
What is pelvic pain?
What are the symptoms related to pelvic pain?
What are the causes of pelvic pain?
How is pelvic pain diagnosed?
How is pelvic pain treated?


Although pelvic pain often refers to pain in the region of women's internal reproductive organs, pelvic pain can be present in either sex and can stem from multiple causes.

Here are further guidelines.

Quiz relevant to topic.
Pain Medicine Examination
Sample Questions September 2013 The following 45 questions are samples of questions that have appeared on previous Pain Medicine examinations.

1. A 55-year-old man has severe pain on gentle touching of the arm. Six months ago, the median nerve was damaged during creation of an arteriovenous fistula for dialysis. Which of the following terms best describes this phenomenon?
(A) Allodynia
(B) Hyperalgesia
(C) Hyperpathia
(D) Hypersensitivity
(E) Hypesthesia

2. Which of the following receptors mediates the windup phenomenon?
(A) 5-HT1D
(B) Mu-opioid
(C) Nicotinic
(D) N-methyl-D-aspartic acid (NMDA)
(E) Vanilloid

3. An 8-year-old girl is scheduled for a caudal injection for treatment of pain after a ureteral reimplantation. Which of the following ligaments will be traversed by the needle immediately prior to entering the caudal canal?
(A) Interspinous
(B) Multifidus
(C) Sacroanal
(D) Sacrococcygeal
(E) Sacrospinous

4. A 25-year-old janitor has severe back pain and is unable to stand up straight after he fell at work. On physical examination, he has positive pain behavior but no neurologic deficits. A surveillance video from the insurance company shows he continues to play baseball with friends. Which of the following is the most likely diagnosis?
(A) Conversion disorder
(B) Factitious disorder
(C) Hypochondriasis
(D) Malingering
(E) Somatization disorder with psychological factors

5. Which of the following conditions is most likely to result from prolonged use of high doses of meperidine to treat pain in a patient with renal failure?
(A) Hyperthermia
(B) Peripheral neuropathy
(C) Progressive pruritus
(D) Seizures
(E) Tolerance to constipation

6. Compared with an epidural dose of morphine, a single epidural dose of fentanyl produces which of the following?
(A) Higher incidence of respiratory depression
(B) Less segmental spread of analgesia
(C) Longer duration of action
(D) More intense pruritus
(E) Slower onset of analgesia

7. A 70-kg, 60-year-old man with hypertension and chronic renal failure is unresponsive and apneic. Two days ago, he began administration of patient-controlled analgesia (PCA) with morphine at a continuous rate of 1 mg/hr with a dose of 2 mg and a lockout interval of 10 minutes. The dose of morphine totaled 60 mg over 48 hours. Which of the following is the most likely cause of the patient's condition?
(A) Accumulation of morphine-6-glucuronide
(B) Accumulation of unmetabolized morphine
(C) Brainstem ischemia caused by hypotension
(D) Mechanical failure of the PCA device
(E) Progressive uremia

8. A 38-year-old woman with breast cancer is switched from oral morphine 90 mg every eight hours to morphine 1 mg daily via percutaneous intrathecal catheter because of excessive sedation. Thirty-six hours later, she has confusion, diaphoresis, and tachycardia. Which of the following is the most likely cause of her symptoms?
(A) Increased intracranial pressure
(B) Meningitis
(C) Metabolic abnormalities
(D) Opioid withdrawal
(E) Sepsis

9. A 47-year-old man requires analgesia in the ICU after sustaining fractures of both femurs in a motor vehicle collision. He has developed hepatorenal syndrome. Which of the following drugs is most appropriate for pain control?
(A) Buprenorphine
(B) Codeine
(C) Fentanyl
(D) Meperidine
(E) Morphine

3 10. A 70-year-old man with early Parkinson's disease is being treated with intravenous patient-controlled analgesia (PCA) with morphine following total knee arthroplasty. His pain is well controlled, but he has nausea. Which of the following is the most appropriate antiemetic for this patient?
(A) Droperidol
(B) Metoclopramide
(C) Ondansetron
(D) Prochlorperazine
(E) Promethazine

11. A 32-year-old woman is referred to a pain clinic to receive analgesic therapy for chronic, intractable low back pain. She has a 15-year history of major depressive disorder, in remission, for which she receives phenelzine. Which of the following medications is most likely to cause serotonin syndrome in this patient?
(A) Gabapentin
(B) Lidocaine
(C) Morphine
(D) Naproxen
(E) Tramadol

12. A 40-year-old patient has difficulty falling asleep at night due to chronic myofascial pain. Which of the following is most appropriate for the treatment of this patient?
(A) Amitriptyline
(B) Combination of butalbital, caffeine, aspirin
(C) Combination of oxycodone and acetaminophen
(D) Fluoxetine
(E) Triazolam

13. A 76-year-old woman has intense paroxysms of unilateral electric-shocklike pain in the face that is usually triggered by brushing her teeth or washing her face. Which of the following is the best initial management?
(A) Administration of baclofen
(B) Administration of carbamazepine
(C) Microvascular decompression of the trigeminal nerve root
(D) Radiofrequency lesioning of the gasserian ganglion
(E) Trigeminal nerve block

14. A 35-year-old woman who has had fibromyalgia for the past 10 years is undergoing evaluation with a new physician. She has been taking alprazolam daily to decrease muscle tension, emotional distress, and anxiety, but her prescription ran out four days ago. She is anxious, irritable, and tremulous. Examination shows tachycardia and diaphoresis. Which of the following is the most appropriate pharmacotherapy for this patient?
(A) Buspirone
(B) Diazepam
(C) Flumazenil
(D) Paroxetine
(E) Propranolol

15. A 62-year-old woman with chronic pain is referred to a pain specialist for consultation. The patient has been taking an undetermined amount of oxycodone for the past four months and the referring physician is concerned that the patient is addicted to pain medications. When the pain specialist goes to see the patient in the hospital room, her adult son is in the room. Which of the following is the most appropriate action by the physician in terms of conducting a patient interview?
(A) Ask the patient whether she would like the son to stay during the interview
(B) Ask the son to leave the room and proceed with the interview
(C) Return later when the patient is alone
(D) Tell the son that he can stay in the room as long as he does not interfere with the interview

16. The ilioinguinal nerve lies
(A) adjacent to the anterosuperior iliac spine
(B) inferior to the inguinal ligament
(C) medial to the femoral artery
(D) superficial to the external oblique muscle
(E) within the inguinal ring

17. A 29-year-old man has had low back pain since falling from a ladder at work and sustaining a coccygeal fracture four months ago. His pain is constant and most severe with sitting. The pain is most likely to be blocked by local anesthetic injection at which of the following sites?
(A) Anococcygeal ligament
(B) Ganglion impar
(C) Pudendal nerves
(D) S4-5 nerve root
(E) Superior hypogastric plexus

18. A 27-year-old woman comes to the pain clinic because of pain in the shoulder and suprascapular region one month after undergoing modified radical neck dissection for papillary carcinoma of the thyroid. Examination shows decreased sensation to light touch over the superior aspect of the right trapezius and scapular winging with resisted shoulder abduction. Which of the following nerves was most likely transected during the procedure?
(A) Axillary
(B) Dorsal scapular
(C) Long thoracic
(D) Spinal accessory
(E) Thoracodorsal

19. A patient has palpitations, flushing, and headache after undergoing gingival injection of a local anesthetic. Which of the following is the most likely cause of this reaction?
(A) Epinephrine in the local anesthetic
(B) Local anesthetic allergy
(C) Methylparaben reaction
(D) Para-aminobenzoic acid allergy
(E) Vasovagal reaction

20. A 31-year-old man undergoes a right thoracotomy and bleb resection with combined general and thoracic epidural (T4-5) catheter anesthesia. Following the procedure, pain is controlled with an infusion of 0.1% bupivacaine and fentanyl 5 µg/mL at 6 mL/hr via the epidural catheter. Twenty hours later, the patient has severe pain at the incision site. Blood pressure is 170/100 mmHg and heart rate is 97 bpm. The most appropriate management at this time is to
(A) administer a bolus of 1.5% lidocaine 4 mL through the epidural catheter
(B) administer 15 mg of ketorolac intravenously every six hours
(C) begin intravenous patient-controlled analgesia with morphine
(D) increase the concentration of bupivacaine to 0.2%
(E) increase the epidural infusion rate of the bupivacaine-fentanyl solution to 8 mL/hr

21. A lumbar sympathetic block is appropriate treatment for each of the following conditions of the lower extremity EXCEPT
(A) Buerger's disease
(B) complex regional pain syndrome type I
(C) frostbite
(D) meralgia paresthetica
(E) phantom limb pain

22. A 42-year-old man has excellent relief of pain five days after undergoing a lumbar epidural injection of methylprednisolone 80 mg diluted to a volume of 10 mL with normal saline. Which of the following is the most common adverse effect of this treatment?
(A) Arachnoiditis
(B) Chemical meningitis
(C) Headache
(D) Hyperglycemia

23. A 44-year-old woman has right hip pain that is reproduced by right leg abduction, external rotation, and palpation of the greater trochanter. Flexion and extension of the hip are not painful. The most appropriate interventional therapy for this patient is injection at which of the following sites?
(A) Hip joint
(B) Ischial bursa
(C) Piriformis muscle
(D) Trochanteric bursa

24. A 30-year-old woman experiences painless shortness of breath 30 minutes after undergoing a stellate ganglion block with 10 mL of 2% lidocaine for treatment of complex regional pain syndrome type I involving the right arm. She had sustained a fracture of the right forearm five months ago. Which of the following is the most likely cause of this symptom?
(A) Anaphylactic reaction to lidocaine
(B) Hematoma
(C) Neurogenic pulmonary edema
(D) Phrenic nerve block
(E) Recurrent laryngeal nerve block

25. Blockade of the superficial cervical plexus is performed at which location?
(A) At the midposterior border of the sternocleidomastoid muscle
(B) In the interscalene groove
(C) Over Chassaignac tubercle
(D) Over the mastoid process
(E) Over the transverse process of C2

26. One percent plain lidocaine 30 mL injected at which of the following sites results in the highest level of systemic absorption?
(A) Brachial plexus
(B) Caudal
(C) Epidural
(D) Intercostal
(E) Subcutaneous

27. During stellate ganglion block, a patient becomes restless, tachypneic, and disoriented immediately after receiving the test dose of local anesthetic. Which of the following is the most likely cause of these symptoms?
(A) Intravenous injection
(B) Phrenic nerve paralysis
(C) Pneumothorax
(D) Subarachnoid block
(E) Vertebral artery injection

28. When using C-arm fluoroscopy to perform a therapeutic procedure, which of the following is the most effective method of preventing unnecessary radiation exposure to the patient? (A) Decreasing the tube distance to the patient
(B) Increasing patient-to-image intensifier distance
(C) Shielding the patient with a lead coverup
(D) Using strict collimation of the beam
(E) Using the shortest beam-on time possible

29. Which of the following is the most likely adverse effect of neurolytic celiac plexus block using 30 mL of 10% phenol in glycerin?
(A) Decreased gastrointestinal motility
(B) Leg weakness
(C) Orthostatic hypotension
(D) Seizure
(E) Urinary retention

30. A 56-year-old man who has pancreatic carcinoma and midepigastric abdominal pain is referred for celiac plexus block. The needles are placed, and their position is confirmed with CT scan. After injection of the first 2 mL of a 0.25% bupivacaine solution containing 1:200,000 epinephrine, the patient's heart rate transiently increases from 68 bpm to 120 bpm. Which of the following is the most appropriate next step?
(A) Administer midazolam 1 mg intravenously
(B) Continue injection of 0.25% bupivacaine with 1:200,000 epinephrine in 3- to 5-mL increments (C) Remove the needles and cancel the procedure
(D) Reposition the needles and repeat the injection of 2 mL of 0.25% bupivacaine with 1:200,000 epinephrine
(E) Wait until the heart rate returns to baseline before continuing the injection of 0.25% bupivacaine without epinephrine

8 31. A 47-year-old woman has had low back pain for the past six weeks that has not responded to treatment with rest, oral analgesics, and physical therapy. MRI of the lumbosacral spine is shown. The image shown reveals which of the following?
(A) Central herniated disk, L4-L5, without compromise of the neural foramina
(B) Central herniated disk, L5-S1, without compromise of the neural foramina
(C) Left-sided herniated disk, L4-L5, with compromise of the neural foramen
(D) Left-sided herniated disk, L5-S1, with compromise of the neural foramen
(E) Normal findings

32. A 60-year-old woman with a chronic S1 radiculopathy has mild bilateral weakness of leg flexion, plantar and dorsiflexion at the ankle, and toe extension. An MRI four months ago showed a small paracentral herniated disk at the L4-5 level. Which of the following is the most appropriate first step in management?
(A) Bilateral selective nerve root injection at the S1 neural foramina
(B) Epidural corticosteroid injection at the L5-S1 level
(C) Repeat MRI
(D) Trial of back extension exercises
(E) Trial spinal cord stimulation

33. A 35-year-old man is referred for evaluation of a six-month history of increasingly severe low back pain that began insidiously without any identifiable trauma. The pain is deep and aching over the low back. He has no pain in his extremities. A discrete lesion is shown on the accompanying MRI within the inferior endplate of the L4 vertebral body. Which of the following is the most appropriate next step?
(A) Obtain a bone scan of the lumbosacral spine
(B) Obtain a complete blood count
(C) Obtain a CT scan of the lumbosacral spine
(D) Obtain an MRI of the lumbosacral spine with gadolinium contrast enhancement
(E) Proceed with initial treatment without further diagnostic evaluation

34. Which of the following is associated with pain arising from the sacroiliac joint?
(A) Paresthesia of the lateral thigh
(B) Positive Patrick's (FABER) test
(C) Sexual dysfunction
(D) Weakness of plantar flexion of the great toe

35. A 14-year-old girl has intermittent bitemporal and occipital headaches that are worse when she is at school. She has undergone counseling at school for depression and anxiety. Which of the following is the most likely diagnosis?
(A) Cluster headache
(B) Malingering
(C) Migraine headache
(D) Temporal arteritis
(E) Tension-type headache

36. A 60-year-old woman comes to the pain clinic because of a two-month history of fever as well as severe, persistent pain over the left side of the jaw and over the left ear. She also has loss of appetite with associated fatigue. Physical examination shows tenderness over the area above her left ear with mild swelling. Erythrocyte sedimentation rate is greater than 48 mm/hr. Which of the following is the most likely diagnosis?
(A) Atypical complex regional pain syndrome
(B) Postherpetic neuralgia
(C) Temporal arteritis
(D) Temporomandibular joint dysfunction
(E) Trigeminal neuralgia

37. A 24-year-old woman has had severe unilateral headaches since menarche. The headaches are preceded by flashing spots before her eyes, are accompanied by nausea with occasional vomiting, and last six to eight hours. The patient reports that she was recently promoted at her job and that her mother and maternal aunt had similar headaches when they were younger. She has been taking oral contraceptives. Although she appears anxious, physical examination is normal. Appropriate management for this patient should include each of the following EXCEPT
(A) administration of acetaminophen with codeine
(B) administration of sumatriptan
(C) avoidance of caffeine-containing beverages
(D) change of her oral contraceptive
(E) maintenance of a strict sleeping schedule

38. A 38-year-old man has had a persistent headache in the frontal and occipital areas bilaterally since undergoing diagnostic lumbar puncture one week ago. The pain is worse when he sits or stands and is relieved when he lies down. The patient is HIV positive and has candidal and cytomegaloviral infections. His CD4+ lymphocyte count is 150/mm3. Which of the following is the most appropriate management of his headache?
(A) Caudal blood patch with donor blood
(B) Caudal blood patch with the patient's blood
(C) Lumbar blood patch with donor blood
(D) Lumbar blood patch with the patient's blood
(E) Lumbar saline infusion

39. A 25-year-old man has pain and paresthesias in the thigh two weeks after being involved in a head-on motor vehicle accident. Current examination shows decreased sensation in the thigh; there is no loss of muscle strength. Which of the following nerves is most likely injured?
(A) Genitofemoral
(B) Ilioinguinal
(C) Lateral femoral cutaneous
(D) Obturator
(E) Saphenous

40. A 28-year-old man has had constant pain in a bandlike pattern at the waistline since sustaining an incomplete transection of the spinal cord at T12 five months ago. Which of the following is the most likely explanation?
(A) Central neuropathic
(B) Mechanical instability
(C) Primary myofascial pain
(D) Psychosomatic pain
(E) Referred visceral pain

41. Which of the following regions is most commonly affected in patients who have acute herpes zoster infection?
(A) Cervical
(B) Lumbar
(C) Sacral
(D) Thoracic
(E) Trigeminal

42. A 35-year-old man who has a 10-year history of HIV infection and intravenous drug use has recent onset of burning pain and dysesthesias in both feet. Nerve conduction studies show a primary sensory symmetric peripheral neuropathy. Administration of which of the following medications is the most appropriate initial step in management?
(A) Dexamethasone
(B) Etanercept
(C) Methadone
(D) Pregabalin
(E) Tramadol

43. A 43-year-old woman is being treated for painful neuropathy in both feet. She is ________, which was caused by a blood transfusion two years ago. Her disease indices are stable, and she is taking appropriate antiviral agents. Acetaminophen/codeine preparations initially helped relieve her symptoms so she could continue to work as a security guard. Which of the following is the most appropriate management of the foot pain?
(A) Discontinue codeine
(B) Initiate antiepileptic therapy
(C) Perform serial lumbar sympathetic nerve blocks
(D) Perform transcutaneous electrical nerve stimulation
(E) Prescribe orthotics

44. A 65-year-old man has had aching in the legs and burning and tingling in the upper and lower extremities for the past two weeks. He has been taking isoniazid for four months for treatment of _________. Which of the following agents is most appropriate for initial treatment of this patient's symptoms?
(A) Amitriptyline
(B) Clonidine
(C) Gabapentin
(D) Pyridoxine (vitamin B6)
(E) Thiamine

45. A 55-year-old woman comes to the pain clinic for a follow-up examination two months after sustaining a fracture of the left wrist. Initial treatment consisted of cast immobilization for six weeks. Since removal of the cast, there has been swelling, sensitivity to touch, erythema, and burning pain. Physical therapy exacerbates the symptoms. A trial of nortriptyline therapy has provided no relief of the pain. Which of the following is the most appropriate next step in management?
(A) Initiate massage therapy
(B) Initiate opioid therapy
(C) Perform peripheral nerve block
(D) Perform stellate ganglion block
(E) No further management is necessary

ANSWERS

1 A
2 D
3 D
4 D
5 D
6 B
7 A
8 D
9 C
10 C
11 E
12 A
13 B
14 B
15 A
16 A
17 D
18 C
19 A
20 A
21 D
22 D
23 D
24 D
25 A
26 D
27 E
28 E
29 C
30 D
31 C
32 C
33 E
34 B
35 E
36 C
37 A
38 D
39 C
40 A
41 D
42 D
43 B
44 D
45 D

Physician’s documentation on record.
Is this acute or chronic pain?

Is this traumatic or nontraumatic pain?

What type of pain is this?

What are causes or specific source of the pain?

What is the best healthcare setting for treatment of this type of pain?

What is the treatment?
Last Updated: November 15, 2020